Download Chapter 7: Economic Growth, Business Cycles, Unemployment, and

Document related concepts

Foreign-exchange reserves wikipedia , lookup

Exchange rate wikipedia , lookup

Currency intervention wikipedia , lookup

Transcript
Chapter 22: Economic Growth, Business Cycles, Unemployment, and Inflation
Questions for Thought and Review
1.
Economic growth is measured by increases in total output and increases in output per
person.
2.
U.S. per capita growth rate of 1.5 to 2.0 percent per year is lower than that of Japan
(4.8 percent per year) and China (2.4 percent per year), close to Western Europe (2.5
percent per year), and Latin America (1.4 percent per year), and higher than Eastern
Europe (1.0 percent per year) and Africa (0.8 percent per year).
3.
Real output is a measure of the total goods and services an economy actually
produces stated in constant prices. Potential output is a measure of the total goods
and services an economy is capable of producing given its resources and institutions.
4.
A representative business cycle is
shown on the right. Each of the
four phases—the peak, downturn,
trough, and expansion are clearly
labeled.
tu
rn
Do
Up
Total Output
Peak
wn
tu
rn
Trough
Time
5.
Besides predicting the recessions that did occur, the index has also predicted many
recessions that did not occur. So the predictions have not been especially accurate.
Predicting recessions is difficult because business cycles have varying durations and
intensities. The leading indicators are called indicators and not predictors because
they're only rough approximations of what's likely to happen in the future.
6.
Reducing unemployment to 1.2 percent in the 1990s is not likely for several reasons.
One is that a low inflation rate seems to be incompatible with low unemployment.
Another is that the 1990s differ from the World War II period when there was an
enormous ideological commitment to the war effort and an acceptance of strong wage
and price controls.
7.
Structural unemployment results from changes in the economy itself, while cyclical
unemployment results from fluctuations in economic activity.
8.
Structural unemployment, because it results from changes in the structure of the
economy, is best studied in the long-run framework. Cyclical unemployment, which
Page 719, Answers to End-of-Chapter Questions
results from fluctuations in economic activity, is best studied in the short-run
framework.
9.
Some economists argue that the unemployment rate undercounts unemployment
because people who have stopped trying to find jobs are considered voluntarily idle
and are not counted as unemployed. Others point out that, because of unemployment
insurance, people often say they are looking for work when they really aren't, and
therefore unemployment is overstated. So there are tendencies both to overestimate
and underestimate the problem.
10.
Okun's rule of thumb states that a 1 percentage point change in the unemployment
rate will cause income to change in the opposite direction by 2 percent. Thus a 2
percentage point rise in unemployment will likely cause income to decrease by 4
percent.
11.
Expectations are central to understanding inflation because the relative price increase
people want must be tacked onto the inflation they expect. Expectations of inflation
play an important role in any ongoing inflation. Expectations can snowball a small
inflationary pressure into accelerating inflation.
12.
While inflation doesn't make the nation any poorer on average, it does have costs. Its
costs include capricious distributional effects, the destruction of the informational
value of prices, and the breaking down of the institutional structure within which
markets work.
13.
Solving inflation often worsens unemployment and slows growth. Similarly, reducing
unemployment by stimulating growth tends to increase inflation. So, in the short run,
there will likely be a relationship. However, once expectations of inflation are built
in, there can be high inflation with high unemployment, so the answer to the question
is no.
Problems and Exercises
1. a. The labor force participation rate is the total number of people employed and/or
looking for work (or the labor force) as a fraction of the population over 16 years old.
In this instance it is 140,910/208,782 X 100 = 67 percent.
b. The unemployment rate is the total number of unemployed as a fraction of the labor
force. In this instance it is 5,689/140,910 X 100 = 4 percent.
c. The employment rate is the total number of unemployed as a fraction of the labor
force. Since the labor force equals the unemployed plus the employed, we know that
in this instance it is (140,910 – 5,689)/140,910 X 100, or 96 percent.
2. a. The index in 2000 is 68/64 X 100 = 106.25.
b. Real output is nominal output/price index X 100 = $300 billion/115 X 100 = $260.9
billion.
Page 720, Answers to End-of-Chapter Questions
c. The change in nominal output = change in real output + change in the price level.
Thus, change in nominal output = 5 percent + 2 percent = 7 percent.
d. The change in nominal output = change in real output + change in the price level.
Thus inflation = 7 percent – 3 percent = 4 percent.
3.
The purpose of life and therefore a desired lifestyle is a complicated issue. To the
extent that work provides a sense of self-worth and identity, complete idleness is not
desirable. This, however, is a normative question. Unemployment within our culture
and set of institutions is a measure of aggregate well-being to the extent that
employment provides a sense of well-being and sufficient income to support a
desired lifestyle. The Eloi are all unemployed, but if they can support their desired
lifestyle with the work of machines, unemployment is not so bad. This example
shows that unemployment must be understood within this broader framework-its
meaning is specific to a set of institutions and a culture.
4. a. Possible explanations include Japanese cultural emphasis on tradition, honor and
loyalty. In Japan, firms are less willing to lay off workers in times of excess supply
and workers are less likely to change employers in search of higher compensation.
Another explanation is the nature of Japanese production. One could suggest that
Japanese production does not rely on a changing base of skills so that the skills of
workers always match the skills demanded of a particular firm.
b. It is impossible to say which is better. Each needs to be judged within the broader
system of the economy. This requires far more knowledge than is required for this
book.
c. The answer to this question depends upon the distribution of layoffs and hires in each
of the economies. If layoffs in Japan were unavoidable and occurred among lower
ranking employees, the average tenure of Japanese employees would rise. If instead
the elderly were asked to retire, the average tenure would decline. In the U.S.,
assuming that new hires came from those just entering the labor force, the average
tenure in the U.S. would most likely decline.
5. a. 5.3%
b. 5%
c. $.02 billion d.2.1
e. 5.3%
Web Questions
1. a. According to the White House’s figures, prices rose by 3.2% in the first quarter of
2000, while the unemployment rate, after seasonal adjustment, stood at 4.1% in
March 2000.
b. According to the Livingston Survey, prices were forecasted to rise by 2.9% in 2000,
while the unemployment rate was to remain steady at 4.2%.
c. Given the recent interest rate increases by the Fed, we would have deduced that
prices had risen by more than what was forecasted. However, due to the growth of the
economy, we would have expected unemployment to have fallen rather than risen.
However, we should keep in mind that these figures are approximates.
Page 721, Answers to End-of-Chapter Questions
Real GDP
Billions of 1996 Dollars
2. a. The answers to this question will
9500
depend upon the current state of
9000
the economy. The graph is
shown on the right. The copy of
8500
the Economic Report of the
8000
President for Spring 2000 did
7500
not include quarterly data back
7000
to 1989, so we looked on BEA’s
peak
web site at www.bea.doc.gov to
6500
89 90
find it. The peak and trough are
marked.
b. The economy is currently in an
expansion.
c. It has been in an expansion for 36 quarters.
d. The last recession was 36 quarters ago in 1990.
trough
91
92
94
95
96
97
99
Year
Chapter 23: National Income Accounting
Questions for Thought and Review
1.
It depends on whether more foreign businesses and individuals conduct business in
the country relative to domestic businesses and individuals. If more foreign
businesses and individuals conduct business in the country relative to domestic
businesses and individuals, then GNP will be greater than GDP.
2.
If you add up all transactions you will include intermediate goods, so the amount will
far exceed GDP, which is the measure of final output within a country.
3.
A stock concept is the amount of something at a given point in time. A flow concept
has a time period associated with it. A stock is the amount of water in a reservoir; a
flow is the amount of water that flows over Niagara Falls every hour.
4.
The aggregate value added at each stage of production is by definition precisely equal
to the value of final sales. Thus the value-added rate should also be 15 percent.
(Technical note: This is assuming the value added tax is an income-based rather than
a consumption-based value added tax.)
5.
If the United States introduced universal child care, GDP should increase because
some child care provided at home would then become market transactions. The
welfare implications of that rise will depend on how society views this change. For
example, would the quality and amount of child care increase? Would there be fewer
neglected and abused children? Would an increase in the number of well-cared-for
children eventually mean lower unemployment, more productive workers, and higher
wages?
Page 722, Answers to End-of-Chapter Questions
6.
NDP is actually preferable to GDP as the expression of a country's domestic output
because it takes depreciation into account. Depreciation is a cost of producing goods.
However, measuring true depreciation is difficult because asset values fluctuate, and
so GDP rather than NDP is generally used in discussions of economic activity.
7.
Employee compensation is the largest component of national income for most
countries.
8.
Transfer payments are not included in national income, and so nothing directly would
happen to it.
9.
Personal income differs from national income because it measures the income
actually received by households as opposed to the income they earned. Thus
unearned income (transfer) is added to personal income, and earned income not
received (corporate retained earnings, corporate income taxes, social security taxes)
is subtracted from personal income to arrive at national income.
10.
The difference between domestic personal income and national personal income is
the addition of net foreign factor income to domestic personal income.
11.
GDP does not measure happiness nor does it measure economic welfare. GDP
measures economic activity. Economists talk about GDP because it is measurable
and they need something to talk about. Moreover, GDP figures are used to make
comparisons of one country's production with another country's and of one year's
production with another year's. Besides, GDP does have some relation to happiness.
Problems and Exercises
1. a.
b.
c.
d.
GDP is the sum of the value added by the three firms: 550 + 1850 + 950 = 3350.
A 10 percent value-added tax would generate (. 10)(3350) = $335 of revenue.
A 10 percent income tax would generate the same revenue.
A 10 percent sales tax on final output would generate (.10)(1000 + 2100 + 1000) =
$410 of revenue.
2. a.
b.
c.
d.
e.
f.
GDP should fall as non-market transactions increase.
GDP would not change.
GDP would rise by the broker's commission.
GDP would not change.
GDP would not change.
GDP would rise.
3.
Students can search on the Internet or call the embassies of each country to find this
information.
4.
GDP = C + I + G + (X - M) = 500 + 185 + 195 + 4 = 884.
Page 723, Answers to End-of-Chapter Questions
GNP = GDP + net foreign factor income = 884 + 2 = 886.
NNP = GNP - depreciation = 886 - 59 = 827.
NDP = GDP - depreciation = 884 - 59 = 825.
NI = NNP - indirect business taxes = 827 - 47 = 780.
PI = NI + transfers from government - corporate retained earnings (undistributed
corporate profits) - corporate income taxes - social security taxes = 780 + 72 - 51 - 64
- 35 = 702
DPI = PI - personal income tax = 702 - 91 = 611
5. a. The expenditure approach:
GDP = C + I + G + (X - M) = 700 + 500 + 300 + 275 = 1775
GNP = C + I + G + (X - M) + net foreign factor income = 700 + 500 + 300 + 275 + (3) = 1772.
The income approach:
National income = wages + rents + interest + profits + proprietor's income: 972 + 25
+ 150 + 500 = 1,647. GNP = NI + indirect business taxes + depreciation = 1,647 +
100 + 25 = 1772.
GDP = GDP - net foreign factor income = 1775.
b. NDP = GDP - depreciation = 1775 - 25 = 1750.
NNP = GNP - depreciation = 1772 - 25 = 1747.
National income = wages + rents + interest + profits + proprietor's income: 972 + 25
+ 150 + 500 = 1,647.
c. PI = NI + transfers - corporate income taxes - undistributed corporate profits - social
security contrib. = 1647 + 0 - 215 - 60 - 0 = 1372.
d. DPI = PI - personal taxes = 1372 - 165 = 1207.
6. a. GDP = C + I + G + (X - M) = 485.
GNP = GDP + Net foreign factor income = 488.
NI = Comp. + rent + profits + net interest = 448.
NNP = NI + indirect business taxes = 475.
NNP = NDP + net foreign factor income: NDP = 472.
b. Depreciation = GDP - NDP = 13.
c. GDP = C + I + G + (X - M) = 480.
GNP = GDP + Net foreign factor income = 483.
NI = Comp. + rent + profits + net interest = 459.
NNP = NI + indirect business taxes = 486.
NNP = NDP + net foreign factor income: NDP = 483.
Depreciation = GDP - NDP = -3.
7. a. 1996: 7813; 1997: 8146.2; 1998: 8496.6; 1999: 8848.9.
b. Since inflation was positive in every year, nominal GDP growth always exceeded real
GDP growth in each of the last three years.
c. It is impossible to say in which year society's welfare increased the most. GDP
measures market activity, not welfare.
Page 724, Answers to End-of-Chapter Questions
Web Questions
1.
a.
b.
c.
d.
The GDP for the last quarter of 1999 was 9,507.9 billion dollars.
Consumption = $6434.1 billion; Investment = $1675.8 billion; Government
Consumption and Investment = $1688 billion; Net exports = -$290.1 billion.
The nominal GDP for the last quarter of 1999 was 9,507.9 billion dollars, while the
real GDP for the same quarter was 9,037.2.
GDP rose by 9.4% in the first quarter of 2000. However, the real GDP rose by 7.3%.
Thus, the increase in the aggregate price level was responsible for 2.1% increase in
the GDP.
The change in consumption contributed the most to the change in GDP. Net exports
changed from -278.2 billion dollars to –290.1 billion dollars and thus became more
negative.
2. a. The economic contributions of household and volunteer work.
b. Crime, depletion of nonrenewable resources, family breakdown and loss of leisure
time. The depletion of nonrenewable resources is the largest of these categories.
c. The GDP has been going up while the GPI has been falling in recent years.
Chapter 24: Growth, Productivity, and the Wealth of Nations
1. a. Answers will vary. Students can do some research about developing countries either
at the library or on the Internet to find out about what it is like to live in a developing
country.
b. Answers will vary. Answers will mention the lack of some products such as cars, airconditioning, dishwashers, TVs, the Internet as well as the need to work longer hours
to provide for the goods that did exist. Figure 24-1 supports this.
2. A person living in 1910 is most likely to work more to buy a dozen eggs than the person
living in 1990. The reason is that since 1910, the United States income has been rising,
on average, by more than the growth in the population. This means that the income per
person has gone up since 1910. Thus the person living in the 1990 has a higher income
than the person living in 1910 and so is likely to work less to buy the dozen eggs.
3. Specialization and division of labor allow a country to take advantage of its comparative
advantage. Thus the individual country can concentrate, (i.e. specialize on the production
of goods in which it has a comparative advantage) and trade them with the goods for
which they do not have a comparative advantage. Hence free trade will, in general,
benefit the participating countries.
4. An increase in savings in the United States can lead to an increase in investment. As
investment in the economy increases, total output in the economy will increase and this
Page 725, Answers to End-of-Chapter Questions
would raise people’s income. As a result, the standard of living will be higher. But if a
politician is to implement policies to encourage savings, it might inhibit aggregate
spending in the economy and so income would not increase and so the policy would not
be able to meet its objective of raising the standard of living.
5. The three types of capital are: i) physical capital ii) human capital and iii) social capital.
Physical capital is the buildings and machines that are available for the production
process; human capital is the workers’ skills that are embodied in them through
education, experience and on-the-job training i.e. through people’s knowledge. Social
Capital is the habitual way of doing things that guides people in how they approach
production in the economy.
6. Two ways in which growth through technology differs from growth through the
accumulation of physical capital are:
i) Accumulation of physical capital increases output by simply increasing the
amount of the capital available for production whereas technology increases
output by making the existing capital more efficient and thereby increasing
the return per available capital.
ii) Technology can also change the types of goods people buy in an economy by
introducing new types of product whereas accumulation of physical capital
does not make such a change.
7. The two actions governments can take to promote the development of new technologies
are:
i) create patents and protect property rights
ii) implement policies to provide funding for research.
8. Thomas Malthus based his prediction that population growth would exceed the growth in
goods and services on the law of diminishing marginal productivity of labor. But his
prediction did not come true because labor has become more efficient as a result of
education and technological progress, which has increased the output per worker.
9. If individuals suddenly needed more food to subsist, the subsistence line would rotate to
the left. Population would decrease and output per worker would fall as in shown in the
graph below.
Page 726, Answers to End-of-Chapter Questions
Subsistence level of
output
Output
Production
function
L1
L*
Labor
10. Positive externalities is the positive effect due to technological change on others not
taken into account when the decision to invest in research was taken. Positive
externalities occur when a technological gain in one sector of production gives other
sectors of production new ideas on how to change what they are doing, which gives other
people new ideas. Thus the effect of the technological change is magnified due to the
spillover effect.
11. The three ways in which growth can be undesirable are:
i) Growth may contribute to increased pollution.
ii) Growth gas changes traditional cultures with beautiful handiwork, music and
dance into cultures of modern gadgets.
iii) Some argue that the average working hours has increased because of growth
in the economy.
12. Credentialism can lead to the employment of workers who have the necessary degree but
do not have the required skills for the job. Thus the average return from the person as a
worker would be less and so the production process he or she is involved would become
less efficient.
13. To compete with the others, the producer has an incentive to innovate i.e. to invest in
technological advance so that he can increase his efficiency and so take a lead among his
competitors. But technological innovation is usually associated with a high cost initially
and the producer who has advanced his production process cannot afford to impose a
high price on his product due to competition in the industry and so he cannot sustain his
technological improvement.
14. The answer may vary depending on the students’ perceptions.
15. Communities are willing to give tax relief to new-technology firms located in their
community because it would enable them to survive during the initial stages when the
start-up costs are very high. If these companies survive, they would be able to provide
Page 727, Answers to End-of-Chapter Questions
employment opportunities to the community members and thereby play a role in
increasing the community’s total income and standard of living.
Problems and Exercises
1.
Using the rule of 72, we know that it would double in 36 years.
2.
Constant returns to scale refers to the relationship between increases in all inputs and
output. In this case, only one input rose, so we cannot make any conclusion about
returns to scale.
3.
Nepal: 65.5 years; Kenya: 42.4 years; Singapore: 10 years; Egypt: 18.5 years.
4.
a. -2.7%
b. 7.6%
c. 1.8%
d. 2.8%
5. a. While developing an educational system for a developing country, one might
emphasize on technical education especially in high school level. The reason for this
is that most students in the underdeveloped countries cannot afford to pursue higher
education and stop at the high school level. Therefore, if high schools focus a little
more on the technical aspect, then these students would be able to garner some
technical knowledge to be efficient in this world of rapid technological change.
b. Because the United States is a rich country, it can afford a more eclectic educational
system that teaches less directly relevant aspects of knowledge.
6. a. The borrowing circle probably would not work in the United States, because the
strong social forces that exists in Bangladesh that eliminate the need for collateral do
not exist in the United States. Perhaps there are some minority groups that do not
have the necessary collateral to get loans in the traditional way but whose culture
could provide the social forces to make repayment of loans more certain.
b. A possible modification of the program would be to require proof that the
“traditional” methods of financing are not open. This would limit the program to
those who have few options, but have a good business plan and intention to repay.
Another modification would be to require that the business be maintained in the
neighborhood where the cosigners live. This would maintain the social forces that
ensure repayment.
c. Minorities often face the same problems because they do not have adequate assets for
collateral necessary to gain traditional financing. They also may face discrimination
by banks and venture capitalists. Nevertheless, they may have good business plans
and an intention to repay.
7. a. It exhibits decreasing marginal productivity.
b. If the population is at L1, it will grow. The surplus food is shown by Q1 – S1.
Page 728, Answers to End-of-Chapter Questions
Subsistence level of
output
Output
Production
function
Q*
Q1
S1
L1
L*
L2
Labor
c. At L2 population is declining because there is not enough food to go around.
d. The intersection of the two curves gives the level of population, L*, at which the
economy is in a steady state. It is a steady state because there is no surplus or
shortage—there is just enough to keep people at their subsistence level.
Web Questions
1. This information was found in the World Development Indicators files.
a. Students have to subtract the population growth from the average annual growth in
output. Kenya = 0.1 %; Mexico = -0.1% – 1.6; United States = 2.1; Japan = 2.7;
b. Kenya = 3.1%; Mexico = 1.9%; United States = 1.0%; Japan = 0.4;
c. Kenya = 19.7% ; Mexico = 23.1% ; United States = ; Japan = 32.3%;
2.
This question was answered according to the Index of Economic Freedom, 2000. The
ratings for the top two are Hong Kong = 1.3; Singapore = 1.45. The ratings for the
bottom two are North Korea = 5.0 and Iraq = 4.90.
a. The top two are countries where there is the greatest absence of government coercion
or constraint in the following spheres beyond what is necessary for the citizens’
protection: production, consumption and distribution.
b. Using figures from 1998, the GDP per capita growth rates were as follows: Singapore
= 0.4 %; Hong Kong = -7.9% ; North Korea = -6.8% ; Iraq = not available. As can be
seen, there does not seem to be a definite relationship between the two factors. Even
if one compares these GDP per capita growth rates with the Index of Economic
Freedom for 1998, one cannot make any definite conclusions. The relationship
between economic freedom and growth may be more visible over longer time
comparisons.
3.
The term of a patent is 20 years although exceptions can be made for, say
pharmaceuticals.
a. Shortening the length of patents means that the new product or the technological
innovation will become “common knowledge” soon. As such, it will be available to
other sectors of production that might use it or might develop new ideas from it.
b. If the length of the patent if shortened, then there is reduced incentive to spend on
Page 729, Answers to End-of-Chapter Questions
R&D and technology since your funds are indirectly being used to provide new
technologies to other sectors.
Chapter 25 Aggregate Demand, Aggregate Supply, and Modern Macroeconomics
Questions for Thought and Review
1.
The central difference between activist and laissez-faire economists is their differing
views about whether the economy is self-regulating. Laissez-faire economists
(Classicals) believe the pricing mechanism will bring the economy to an equilibrium
(potential output and full employment) while activist economists (Keynesians) do not
share that belief.
2.
Classicals felt that if the wage was lowered, the depression would end. They saw
unions as preventing the fall in wages, and they saw the government lacking the
political will to break up unions.
3.
Five factors that shift the AD curve are: changes in foreign income, changes in
expectations, changes in exchange rates, changes in the distribution of income, and
changes in government aggregate demand policy.
4.
Say there is a rise in the price level. That would make the holders of money poorer
(the wealth effect). It would also reduce the real money supply, increasing the interest
rate (the interest rate effect). Assuming fixed exchange rates, it would also make
goods less internationally competitive (the international effect). All these three
account for the quantity of aggregate demand decreasing—decreasing spending as the
price level rises. These initial increases are then multiplied by the multiplier effect as
the initial spending reverberates through the economy.
5.
Two factors that shift the AS curve are changes in productivity and changes in input
prices.
6.
If the economy is in short-run equilibrium below potential output, there will be
downward pressures on the price level. The aggregate supply curve will shift down
and the price level will fall. This will set the wealth, interest rate, and international
effects in motion, increasing the quantity of aggregate demand and thereby bringing
the economy into long-run equilibrium at potential output.
7.
This implies that productivity is increasing significantly. If computers are a large
portion of the economy, and wages do not rise by the full amount of the productivity
increase, the result will be to lower the AS curve. It can also shift out the potential
output curve to the right, increasing equilibrium potential output and lowering the
price level.
Page 730, Answers to End-of-Chapter Questions
8.
Yes; they would emphasize the inherent value of the program, rather than discussing
the program's effect on aggregate demand. This is because programs that increase
aggregate demand when the economy is close to potential will ultimately lead to
inflation and little increase in real output.
9.
As can be seen in the following diagram, a large increase in potential output causes
downward pressure on the price level from P0 to P1. As the price level shifts down
the output level increases from Y0 to Y1.
Price
Level
P0
Potential Output
Curve
E
AS 0
D
P
AS1
1
AD
Y0
10.
Y
1
0
Real Output
The simple model abstracts from a number of important issues such as the problem of
estimating potential income. Without knowing potential income we cannot know
whether expansionary or contractionary policy is called for.
Problems and Exercises
1. a.
b.
c.
d.
The AS curve will shift up since wages rise by more than the rise in productivity.
The AS curve will shift down since productivity rises by more than the rise in wages.
The AS curve will shift up since wages rise and productivity declines.
The AS curve will not shift since the wage increase is exactly offset by a productivity
increase.
2. a. Keynes used models not in a mechanistic way, but in an interpretive way. He was a
Marshallian who saw economic models as an engine of analysis, not an end in
themselves.
b. It fits in nicely with the "other things constant" assumption since the policy relevance
follows only when one has eliminated that assumption and taken into account all the
things held at the back of one's mind.
c. It definitely was primarily in the art of economics since the above method is the
method used in the art of economics.
3. a. The AD curve will be steeper because a change in the price level will be offset by a
change in the exchange rate eliminating the international effect on the AD curve.
Page 731, Answers to End-of-Chapter Questions
b. The AD curve will become steeper if a fall in the price level doesn't make people feel
richer since the fall in the price will not cause them to increase their expenditures.
This is an example of the wealth effect not working.
c. The AD curve will be steeper if a fall in the price level creates expectations of a
further fall in the price level (it may even be backward bending) since the fall in the
price level will cause people to shift expenditures further out into the future.
d. Assuming poor people consume more than rich people, the AD curve will shift to the
right.
e. The AD curve will shift to the right by a multiple of 20.
f. The AD curve will shift to the left by a multiple of 10.
4. a. An increase in the availability of inputs will shift the potential output curve to the
right.
b. A civil war will presumably destroy productive capacity or otherwise halt production
and cause a shift in the potential output curve to the left.
c. To the degree that the rise in oil prices results in an overall rise in the price level, this
will shift the AS curve up. Otherwise, other relative prices will decline to offset the
rise in oil prices and the AS curve will not shift at all.
d. If wages become flexible and were previously fixed and aggregate demand increases,
the AS curve will shift up as wages rise.
5. a
We would suggest that the rise in oil prices will shift the AS curve up and the drop in
world income will shift the AD curve in, causing equilibrium income to fall even
more below potential (to point B in the graph below).
b. We might suggest expansionary fiscal and monetary policy to shift the AD curve out
(from AD0 to AD2) and bring equilibrium income to its potential. We would caution
the government about the possible inflationary consequences, but since the economy
is significantly below potential, we would argue that it is a risk worth taking.
Potential Output
Curve
Price
Le ve l
P
B
AS 1
1
A
P
AS 0
0
AD
AD 2
0
AD
1
Y
1
Y0
Re al Output
c. A real business cycle economists would say that the actual level of the economy is
the best estimate of its potential income. He would suggest that the policy would be
inflationary because it is not affecting the real supply-side issues. If the economy
Page 732, Answers to End-of-Chapter Questions
were left on its own, the AS curve will shift down below AS0 until output reached its
potential
Web Questions
1.
The CPI has been rising. Since there is inflationary pressure in the economy, we
would conclude that the economy is in the intermediate range. We would therefore
support tighter monetary policy.
2. a.
GDP
6,591.8
6,707.9
6,676.4
6,880.0
7,062.6
7,347.7
7,543.8
7,813.2
8,144.8
8,495.7
8,848.2
Price Level
83.3
86.5
89.7
91.4
94.0
96.0
98.1
100
101.9
103.1
104.6
105
Price level
Year
1989
1990
1991
1992
1993
1994
1995
1996
1997
1998
1999
100
95
90
85
80
00
65
00
70
00
75
00
80
00
85
00
90
Real output
(billions of real dollars)
b. See the graph on the right of the table.
c. Since the curve involves shifts in both the AS curve and AD curves, all we can say is
that these are points of equilibrium given certain assumptions. It is neither an AD nor
an AS curve. In order to draw one or the other simplifying assumptions must be made
regarding what is held constant.
3.
Even though the economy is still growing consumer confidence has been falling
lately suggesting that the AD curve may start to shift to the left. If this happens,
output will decline as shown in the graph below. We have drawn it so that output
declines to potential, but if the economy falls below potential, the AS curve will shift
down, bringing equilibrium output back up at a lower price level.
Page 733, Answers to End-of-Chapter Questions
Potential Output
Curve
Price
Level
P
B
A
AS 0
0
AD
0
AD
1
Y
1
Y0
Real Output
Page 734, Answers to End-of-Chapter Questions
Chapter 26: The Multiplier Model
Questions for Thought and Review
1.
If planned expenditures are below actual production, income will decline. Here’s
how: when planned expenditures are below actual production, firms will see that
their inventories are building up faster than they’d like. In response, they cut
production. As production falls, so does income. Consumption falls by a fraction of
the decline in income leading to a further decline in planned expenditures. This
process continues until planned expenditure equals actual production.
2.
Inventories are building up at levels of output above equilibrium because at levels
above equilibrium, planned expenditures are below actual production. People are not
buying all that is produced.
3.
The aggregate expenditures curve shifts down by the decline in autonomous
expenditures.
4.
The AE curve becomes steeper when the marginal propensity to consume increases.
Equilibrium income rises.
5.
If savings were immediately translated into investment, the size of the multiplier
would be infinite since leakages from the economy would be zero. However,
autonomous expenditures would no longer exist. In short, under these conditions the
multiplier model would break down.
6.
Shocks to aggregate expenditures are any sudden changes in factors that affect C, I,
G, X, or M. This includes consumer sentiment, business optimism, foreign income,
and government policy. It is possible that people could change their marginal
propensities to consume and save, and this could also have an effect on the economy.
7.
The effects on the economy of this invention would be manifold and in many ways
unpredictable because such major shocks have social, institutional and political
effects, as well as economic effects. The obvious effect is that the demand for the pill
would likely be tremendous (after people were sure it was safe), and so production of
the pill would gear up to meet the demand. Market structure and pricing decisions
will play a big role in determining the new effect of the change. Alternative forms of
transportation would suffer decreases in demand (cars, mass transit, airplanes, etc.),
and levels of production of those goods and services would adjust, as would
employment in those industries and related industries. Measured GDP might actually
fall.
8.
The circular flow diagram of the economy that would more accurately describe the
multiplier model would include leakages of savings to investment that cause the
Page 735, Answers to End-of-Chapter Questions
diagram to pulsate as the economy continually overshoots equilibrium in response to
shocks to the economy.
9.
A mechanistic model states the equilibrium independent of where the economy has
been or of what people wanted. A mechanistic model is used as a direct guide for
policy prescriptions. An interpretive model is used as a guide that highlights dynamic
interdependencies that is suggestive of the direction of the response of aggregate
output to various policy initiatives.
10.
If there is a delay, it will mean that the initial multiplier effects can be small or nonexistent, and then, suddenly, they become large and fast. Uncertain, changing,
expectations can add to the ambiguity of the model’s result.
Problems and Exercises
If the mpc is .8, then the value of the multiplier
is (1/.2) or 5. If autonomous expenditures are
$4,200, the equilibrium level of income in the
economy is 5 x 4200 = $21,000. This
isdemonstrated in the graph on the right.
Real expenditures
1.
AP
AE
$4,200
$21,00
Real income
Real expenditures
2. a. If the mpc is .66, the value of the multiplier is 3. A decrease in autonomous
expenditures of $20 will likely result in a decrease in income of $60. This is
demonstrated in the graph below.
AP
AE0
AE1
D 20
D 60
3.
Real income
Given the mpc is 0.8 and autonomous investment has risen by 20,
a. Income will increase by 100 (the multiplier is 1/.2 or 5, and 5 X 20 is 100).
b. The multiplier is now only 2 (1/.5), and so the change in investment causes income to
change by 40.
Page 736, Answers to End-of-Chapter Questions
c. The decrease in exports and increase in investment cancel each other out so that
autonomous expenditures in the aggregate are unchanged.
d. See the graphs below. The graph on the left corresponds to (a) and the graph on the
right corresponds to (b). The graph to (c) would show the AE curve not moving at all.
AP
AE0
20
E0
100
AP
AE1
Real expenditures
Real expenditures
E1
AE1
E1
20
E0
40
Re al income
4.
AE0
Real income
Given that the mpc is .6, I = 1000, G = 8000, C = 10,000 and (X - M) = 1000,
a. Y = 10,000 +.6Y + 1000 + 8000 + 1000.
Y - .6Y = 20,000; 0.4Y = 20,000; Y = 50,000. Thus the level of income in the
country is $50,000 (note that Examland seems to be on a U.S. dollar currency
standard).
b. If net exports increase by $2000 income will increase by $5000 (the multiplier is 2.5
or 1/.4).
c. According to Okun's law, a 1 percentage-point change in unemployment will cause a
2 percent change in income in the opposite direction. Thus if income has increased by
$5000, which is a 10 percent increase, then unemployment should drop by 5
percentage points.
d. If the mpc falls from 0.6 to 0.5, the multiplier decreases from 2.5 to 2. The answer for
part a would now be $40,000, the answer for part b would be $4000, and the answer
to part c is that unemployment should rise by 4 percentage points.
Real expenditures
5. a. A likely culprit was a decline in investment
spending, partly due to increased bank
AP
regulation and Federal Resolution Trust
AE0
Corporation scrutiny of loans in the wake of
AE2
the failed S&Ls and liquidity problems of
AE1
commercial banks in the late 1980s and early
1990s. This was commonly known as the credit
crunch, where lower interest rates failed to
increase investment spending in the early
Real income
1990s. This is shown as a shift down of the AE
curve from AE0 to AE1 and a decline in real income.
b. An improvement would be graphically represented by a shift up of the AE curve
shown in the graph as the shift from AE1 to AE2 and a rise in real income. The
improvement occurred most likely as a result of expectations of an improving
economy and further reductions in interest rates increasing consumption and
Page 737, Answers to End-of-Chapter Questions
investment expenditures. Government expenditures did not change much in this
period and probably did not contribute to the economic improvement.
c. President Bush would have had to increase government expenditures significantly to
stop the slowdown, but given the political atmosphere regarding the high deficit, and
debt, it is unlikely he could have done so.
d. President Clinton faced the same political imperatives to decrease the size of the
deficit, so he implemented some policies designed to affect potential output (the
supply side). He lowered some taxes, calling them supply enhancing tax cuts,
changed the composition of government spending calling them supply-enhancing
changes, and raised other taxes (discounting their effects on the economy).
Web Questions
1. a. Consumer confidence slipped in April 2000, after having already declined in March
2000. It is now at 136.9 (1985=100) down from 137.1 in March.
b. Falling consumer confidence means that consumption expenditures, and hence
aggregate demand will fall. The multiplier model says that the decline in national
output will be greater than the fall in aggregate demand.
c. Business confidence rose slightly to 52 in the final quarter of 1999, up from 51 in the
previous quarter. (A reading of more than 50 reflects more positive than negative
responses).
d. They do not match. We would therefore expect inventories to increase in the coming
months since consumption expenditures will fall.
2. a. Manufacturers’ and trade inventories in March 2000 were up 0.3% (±0.1%) from
February 2000. Total durable goods were up 0.3 percent (±0.2%) from February
while total nondurable goods increased 0.4 percent (±0.1%) from February.
b. Rising inventories means that consumer expenditures, and therefore aggregate
demand is declining. The multiplier model tells us that the change in national income
will be greater than the change in aggregate demand. We would therefore expect the
economy to slow down.
Appendix A: An Algebraic Presentation of the Expanded Multiplier Model
1.
We would suggest a decrease in taxes. To determine precisely how much we would
need to determine what the multiplier is. Assuming all other marginal propensities
are zero, the multiplier is 5. The tax cut would initially affect the economy by only .8
times the tax cut, so to increase output by 400, we would decrease taxes by 100. (.8 X
100 X 5 = 400).
2.
We would recommend increasing expenditures by 80.
3.
This makes the multiplier 3.57. This means that we would increase expenditures by
about 112, or cut taxes by about 140.
Page 738, Answers to End-of-Chapter Questions
4.
This makes the multiplier 2.08. This means that we would increase expenditures by
about 192 or cut taxes by about 240.
5.
Making taxes and imports endogenous reduces the size of the multiplier because they
increase the leakages from the expenditure flow.
6.
This would make the multiplier = 1/(1 - c + ct + m - mt). It would be a slightly higher
multiplier. (The difference between the two assumptions is whether we are assuming
government imports.)
Appendix B: The Multiplier Model and the AS/AD Model
AP
AE1
AE0
Price level
Real expenditures
1. a. As shown in the left-hand graph below, an increase in autonomous expenditures
shifts the AE curve up and causes a movement along the AP curve to the right and
results in a higher equilibrium income level twice the shift in the AE curve.
b. As shown in the right-hand graph below an increase in autonomous expenditures
shifts the AD curve to the right by twice the increase in autonomous expenditures.
Since the price level is fixed, Real output increases by twice the rise in autonomous
expenditures as well.
AS curve
AD1
AD0
Re al income
Real output
c. Since prices are somewhat flexible, the rise in expenditures is split between a rise in
prices causing a downward shift of the AE curve that is smaller than the initial
upward shift. The rise in income is less than twice the initial shock. This is shown in
the graph below on the left.
In the AS/AD model, a flexible price means that the shift in the AD curve is split
between increases in the price level and increases in real output. Real output rises by
less than the multiplier times the increase in autonomous expenditures.
2. a. The AD curve will become steeper.
b. An increase in the size of the multiplier does not affect the slope of the AD curve; It
only increases the size of the shifts in the AD curve in response to a shift in
autonomous expenditures.
c. An increase of $20 in autonomous expenditures has no effect on the slope of the AD
curve; it only affects its position.
Page 739, Answers to End-of-Chapter Questions
AP
AE1
AE2
AE0
Price level
Real expenditures
d. A decline in the price level disrupting the financial market will make the AD curve
steeper because it eliminates the price level interest rate effect.
Potential
output
P1
AS1
AS0
P0
AED1
AED0
Real income
Y0 Y1
Real output
Chapter 27: Demand Management Policy
Questions for Thought and Review
1.
Franklin D. Roosevelt's statement was made during the Depression of the 1930s,
which was partly caused by people getting scared and cutting back their spending.
The multiplier then took over and turned that decrease into a much larger decrease.
To get the multiplier working in reverse, and so countershock the economy, the
government would have had to get people to spend more than they wanted to; hence
Roosevelt's attempt to calm people's fears and to encourage them to spend.
2.
In early 2000, the government budget was in surplus and the state of fiscal policy was
contractionary. This policy made sense because many believed the economy was
beyond its potential income and the biggest threat was inflation. Because of strong
consumption and investment spending, most economists felt that a contractionary
fiscal policy was appropriate. (This answer may change as the economy progresses
through the 2000s.)
3.
If the mpc is .5, the multiplier is 2. Every one-dollar increase in autonomous
expenditures will raise income by two dollars. To close a recessionary gap of $200
the government needs to generate $100 of additional autonomous spending. It can
accomplish this by increasing government expenditures by $100.
4. If the economy has a recessionary gap, the following trade policies can be adopted:
i)
an export-led growth policy in which the country lobbies to remove other
countries’ restrictions on its exports.
ii)
Allowing its currency to depreciate which means that the exchange rate of its
currency relative to other currency should fall.
5.
Cutting taxes by $100 has a smaller effect on GDP than increasing expenditures by
the same amount because people don’t spend the entire amount of the tax cut. The
Page 740, Answers to End-of-Chapter Questions
multiplier begins with the increased individual spending resulting from the tax cut, or
the mpc times the tax cut.
6.
To offset a cut in taxes, the U.S. government should implement a policy that reduces
net exports by the mpc times the tax cut such as instituting new or higher tariffs.
7.
Countercyclical fiscal policy is difficult to implement because politically it is difficult
to raise taxes when the economy is doing well (or at any time). Politics, not the needs
of the economy guide tax and spending decisions.
8.
If interest rates have no effect on investment, there would be no crowding out.
Crowding out occurs when the government's sale of bonds to finance expansionary
fiscal policy causes interest rates to rise, choking off private investment.
9.
To design an appropriate fiscal policy, it is important to know the level of potential
income because where the economy is relative to potential income tells you whether
you want expansionary or contractionary policy. Conducting fiscal policy without
having an estimate of potential income would be like driving without being able to
see the road.
10.
Increasing government spending shifts out aggregate demand and thereby increases
income. This makes people better off in the short run and more likely to vote for
them. The exception would be if the economy is beyond potential income and there is
a significant inflation threat.
11.
Increasing taxes shifts the aggregate demand curve in decreasing income and making
people less likely to vote for them. Luckily, for politicians, people tend to have short
memories.
12.
Even though unemployment was high, most economists felt that the cause of the
unemployment was structural, and that the economies were at their potential incomes.
Thus, deficits would only be inflationary.
Problems and Exercises
1. a. If the mpc is .8, the multiplier is 5. Every one-dollar increase in autonomous
expenditures will raise income by five dollars. To close a recessionary gap of $400
the government needs to generate $80 of additional autonomous spending. It can
accomplish this by increasing government expenditures by $80.
b. If the government wishes to achieve the same end by changing taxes, it should
decrease taxes by $100. This will generate $80 of additional autonomous spending.
Again, with a multiplier of 5, this will cause a $400 increase in aggregate income.
c. If there is a marginal tax rate of 0.2 (instead of taxes being exogenous), the multiplier
is [1/(1 - c + (c ( t))] or [1/(1 - .8 + (.8)(.2))], which is equal to 2.77. With regard to
your answers to parts a and b, the government must generate added expenditures of
Page 741, Answers to End-of-Chapter Questions
Real expenditures
AP
Real expenditures
$144.40 to close the recessionary gap, either through an increase in government
spending of that amount (part a) or a decrease in taxes of $180.50 (part b).
d. With a marginal propensity to import of .2 the multiplier changes to [1/(1 - c + ct +
m)] or [1/(1 - .8 +.16 +.2)] which is equal to [1/.56] or 1.79. With this multiplier, to
close a recessionary gap of $400 the government would have to generate $224 in new
expenditures, either through an increase in government spending of that amount or a
cut in taxes of $280.
e. The graph on the left below shows the shift in the AE curve resulting from an
increase in government expenditures of $80 or a reduction of taxes of $100 when the
mpc is .8. The graph on the right shows the shift in the AE curve necessary to close
the recessionary gap when the marginal tax rate is .2. Notice that the slope of the AE
curve is smaller (.64). The AE curve must shift up by more to achieve the same
increase in output. Finally, the AE curve for part d has an even smaller slope (.44)
shown below the first two graphs.
Here the AE curve must shift up by
even more.
AP
AE1
AE0
80
AE1
AE0
144.4
400
400
Real income
Real income
Real expenditures
AP
AE1
AE0
224
400
Real income
2.
Given that income is $50,000, the mpc is .75:
a. To reduce unemployment by 2 percentage points (again, by Okun's rule of thumb)
requires a 4 percent increase in income, which in this case is $2000. The multiplier is
4.0, calculated as [1/(1 - mpc)]. To generate a $2000 increase in income, increase
government spending by $500.
b. If the mpc is .66, the multiplier is 2.94, which means that to generate a $2000
increase in income, government spending would have to increase by $680.27.
Page 742, Answers to End-of-Chapter Questions
c. If the mpc is 0.5, then the multiplier is 2.0, which means that to generate a $2000
increase in income, government spending would have to increase by $1,000.
3. a. In 1995 the unemployment rate fell below the target rate of 6% without generating
inflationary pressures. He was probably changing his estimates to reflect that reality.
b. It would shift the potential output curve out.
c. Using Okun's rule of thumb that for every 1 percentage point rise in the
unemployment rate, income falls by 2%, a 0.5 percentage point decline in the target
unemployment rate would imply a rise in potential income of 1%, or $73 billion.
4. a. President Clinton's policy does not fit well with the multiplier model because with
that model the two goals are inconsistent with one another in the short run. To
increase output and employment using stimulative fiscal tools requires an increase in
the deficit.
b. His policy might have the desired effect if a reduction in government expenditures to
reduce the deficit reduces the interest rate so much and affects expectations positively
to such a degree that their effect on investment and consumption expenditures offsets
the decline in government spending.
c. The reduction in the deficit shifts the AD curve to the left, but the increase in
investment expenditures resulting from the reduction in interest rates shifts the AD
curve to the right by so much that, on net, output rises as shown in the graph below.
Price
Level
Aggregate
Supply
F
P
0
AD
2
AD 0
AD 1
Y1
Y
0
Y2
Real Output
d. I would look at interest rates and investment expenditures to see if the explanation
is correct.
5. a. To eliminate the inflationary gap, the government should undertake a contractionary
fiscal policy. Since the economy is $36 thousand above potential, we would advise
decreasing government spending by $18 thousand.
b. Using Okun’s rule of thumb since income falls by 6%, we would expect
unemployment to rise by 3 percentage points to 8%.
c. The multiplier now becomes 5, so we would advise decreasing spending by $7.2
thousand. We would not change our answer to b.
Web Questions
Page 743, Answers to End-of-Chapter Questions
1. a. The four key economic targets are: low inflation, low level of unemployment, high
and stable level of economic growth, and external balance between imports and
exports.
b. The main policy tools are:
Income tax (rates and allowances): This is the key variable of the fiscal policy and
can be used to alter consumer expenditures.
VAT and other indirect taxes: This indirect tax works like the income tax and can be
used to alter expenditures. It is also an important source of government revenue.
Government expenditure: This is also a part of the fiscal policy. It includes
expenditures on things like defense and highways etc.
Interest rate: This is a part of the government’s monetary policy and can be used to
control variables like the inflation rate.
c. Assuming that the economy is in equilibrium below the potential output, if income
taxes are increased, the resulting fall in aggregate demand will lead to a fall in output
at the same level of prices. Thus there will be lower economic growth, greater
unemployment, but no change in the inflation rate. The model also predicts imports
to decrease.
d. Assuming that the economy is in equilibrium below potential output, the model
predicts that the increase in government spending will lead to greater output and
hence greater economic growth. There will also be less unemployment and more
imports though the inflation rate would remain the same.
2. a. The process starts off by the formulation of the President’s budget for a fiscal year.
The budget documents are then prepared and transmitted to the Congress. The
Congress after reviewing this budget develops its own budget and accepts the
expenditure and revenue bills. The agency managers then execute the budget in the
fiscal year after which information for the actual spending and receipts becomes
available.
b. It takes about two years. For example, for Fiscal Year 2001 (begins October 1, 2000)
the President formulated the budget between February-December 1999, and the data
on the expenditures and receipts became available in October-November 2001.
c. The President and the Congress have to decide upon the discretionary spending,
which accounts for one-third of all federal spending. The remaining two-thirds of all
federal spending, called mandatory spending, is authorized by permanent laws.
Chapter 28: Politics, Surpluses Deficits and Debt
Questions for Thought and Review
1.
It follows from the long-run framework. The long-run framework directs one to avoid
deficits; in the short-run framework deficits are useful if the economy is significantly
below potential.
2.
Two ways the government can finance a deficit is by selling bonds or by printing
money.
Page 744, Answers to End-of-Chapter Questions
3.
It depends. Your cash flow budget is in deficit since your expenditures exceed your
income. However, if you consider your tuition expense an investment in your future
and separate it out as part of your capital budget, then your current account budget
will have a surplus.
4.
There are technical aspects of the deficit that must be understood in order to
undertake a meaningful discussion of the problems deficits and debts pose for
society. Since a deficit is defined as a shortfall of revenues compared to expenditures,
these technical aspects include what you define as revenue and what you define as
expenditure.
5.
Given that the nominal deficit is $300 billion, the inflation rate is 20 percent, and the
debt is $2 trillion, the real deficit is calculated as the difference between the nominal
deficit and the product of the inflation rate and the total debt, or 300 billion - (.20 )( 2
trillion) = (-100) billion. There is a real surplus of $100 billion.
6.
It would not differ; expected inflation does not enter into the determination of the real
deficit.
7.
The structural deficit is greater than the actual deficit because the economy is above
its potential. Assuming expenditures do not change with income, tax revenues are
$50 billion dollars higher than they would be if the economy were at its potential, so
the structural deficit is $150 billion. The passive surplus is $50 billion because tax
revenues are $50 billion higher because the economy is operating above potential.
The structural and the passive deficit add up to the actual deficit.
8.
A structural deficit would exist even if the economy were at its potential level of
income, which would be at full employment, or where the unemployment rate were
equal to the normal rate of unemployment. If an economist believed that the normal
rate of unemployment was 4 percent instead of 6 percent, then the portion of the
deficit considered to be structural would be smaller. Thus, it is Mr. A who should
also say that the structural deficit is $20 billion.
9.
Debt is defined as accumulated deficits, and is also a summary measure of a country's
financial situation. Debt must be viewed in relation to a country's assets, but, like
income and revenues, assets and debts are subject to varying definitions. There's no
unique answer to how assets and debts should be valued, and there are different types
of debt.
10.
Three ways individual debt can differ from government debt are: 1) government is
ongoing, and never needs to pay it back; 2) government can pay off the debt by
creating money; and 3) much of the government debt is internal debt.
11.
No. Financing internal debt causes redistribution; it does not make the society poorer.
Page 745, Answers to End-of-Chapter Questions
12.
Deficits are only a summary measure of the economy. A government can undertake
significant future obligations and therefore get itself into trouble even if it is not
running a deficit.
13.
Debt service is a good measure of whether debt is a problem because interest
payments do not result in additional productive expenditures. Interest payments are
the result of past expenditures.
14.
The Budget Enforcement Act changed the politics of spending and taxing. It forced
Congress to figure out how they would pay for a program at the same time it
instituted the program.
15.
Because of the baby boom in the late 1990s there were many people working and
relatively few collecting Social Security. This caused a surplus in the Social Security
Trust Fund. Since that Trust Fund is part of the government budget, the Social
Security system is a primary reason for the surplus.
16.
Two real solutions are increasing the retirement age and cutting benefits when baby
boomers retire.
17.
It depends. Clearly, there is some tendency for deficit to raise the interest rate,
thereby decreasing investment and hence future growth. However, to the degree that
the government spending is itself productive, not having the deficit could also
decrease future growth. The ultimate effect depends on the relative size of the two
effects
Problem and Exercises
1.
a.
b.
c.
d.
The real deficit is calculated as the nominal deficit less the product of the inflation
rate and the total debt. Therefore,
Real deficit = 220 billion - (.10 X 3 trillion) = (-80) billion
50 billion - (.02 X 1 trillion) = 30 billion
30 billion - (-.04 X 500 billion) = 50 billion
$100 billion surplus + $60 billion = $160 billion surplus.
2. a. Debt service payments are 0.06 times $360 billion = $21.6 billion.
b. The nominal deficit is 160 - ($21.6 + 145) = $6.6 billion.
c. The real deficit equals the nominal deficit ($6.6 billion) less .03 X $360 = $4.2
billion surplus.
3.
Since the real deficit is rising at a faster rate than real growth in the economy, the
deficit/GDP ratio is rising. It can only continue to do so as long as it can sell the
bonds, but at some point the deficit/debt ratio will become so large that creditors will
Page 746, Answers to End-of-Chapter Questions
begin to doubt the country's ability to repay the debt. Then, selling bonds will become
more difficult and eventually impossible.
4.
To make the deficit look as small as possible, we would do the following:
a. Enter government pensions when they become payable, not on an accrual basis.
b. Treat the sale of land as current income rather than spreading it out with the sale of
an asset.
c. Include Social Security taxes as a current revenue because at this time revenue from
Social Security exceeds payment.
d. Count prepayment of taxes as current income instead of reserves for future taxes.
e. Count expenditures on F-52 bombers as capital expenditures and introduce another
cost item of depreciation and allow the F-52 to depreciate as little as possible.
Web Questions
1. a. The system is designed so that there is a direct link between how much a worker pays
into the system and how much he gets back as benefits. Secondly, the social security
program is a way of providing economic security through retirement, disability and
survivor insurance packages.
b. We would be eligible for reduced benefits if we retire early at the age of 62.
However, we will get full-retirement benefits if we retire at the age of 65. These
benefits replace about 40% of one’s pre-retirement income.
c. The following are some of the options available: raising the retirement age, levying
social security taxes on all incomes, creating individual saving accounts for workers
to supplement social security benefits, and investing social security reserves in the
stock market.
2. a. The U.S. first had debt in 1790 when it assumed the Revolutionary War debts of the
Continental Congress.
b. The actual debt at the end of 1997 was $5,369,707 million. This was more than
previous year’s debt of $5,181,934 million. The estimate for 2000 is $5,915,719
million.
c. The U.S. government through the Treasury department does so by selling new bonds,
and refinancing old ones. Printing money would lead to inflation.
Chapter 29: Money, Banking, and the Financial Sector
Questions for Thought and Review
1.
Although financial institutions don't produce any tangible real assets, they are
nonetheless considered a vital part of the economy because of their central role in
transferring savings into investment and in making the real economy more efficient.
2.
Money is to the economy as oil is to an engine because money is a financial asset that
makes the real economy function smoothly by serving as a medium of exchange, a
Page 747, Answers to End-of-Chapter Questions
unit of account, and a store of wealth. Without it the economy comes to a screeching
halt.
3.
The three functions of money are: (1) it serves as a medium of exchange; (2) it serves
as a unit of account; and (3) it serves as a store of wealth.
4.
In order to maintain money's usefulness and to prevent large fluctuations in the price
level, the money issuer, which in the United States is the Federal Reserve Bank, must
issue neither too much nor too little money. To issue money without restraint would
destroy the social convention that gives money its value.
5.
Two components of M2 that are not components of M1 are savings deposits and
small denomination time deposits.
6.
The equation for the simple money multiplier is 1/r; the equation for the approximate
real-world multiplier is 1/(r+c). Since c is positive the simple multiplier is larger.
7.
Money doesn't have to have any inherent value to function as a medium of exchange.
All that's necessary is that everyone believes that other people will accept it in
exchange for their goods. This is the social convention that gives money value.
8.
If the U.S. government were to raise the reserve requirement to 100 percent, the
interest rates banks pay to depositors would decrease and possibly even become
negative (you'd have to pay to have the bank handle your money), because significant
opportunities for profitable loans would be lost.
9.
A benefit of government guarantees is that it prevents inappropriate panic about a
bank’s ability to pay back its obligations; a cost of government guarantees is that it
prevents appropriate panic about a bank’s ability to pay back its obligations.
10.
What brought the S&Ls down was bad loans, particularly in real estate. The reasons
that S&Ls made those bad loans are more complex. Government deregulation in the
1980s expanded the kinds of loans S&Ls could make and the ways they could
compete for deposits. Due to moral hazard and perverse incentives (government, not
the bank managers, would have to pay if the S&L went down), S&Ls made risky
loans and paid high interest on their deposits. When the real estate market soured,
S&Ls' net worth crumbled, and the government had to step in to bail out depositors.
11.
Panics occur when depositors lose faith in a bank and all try to take their funds out at
once. The U.S. banking system is less susceptible to panics because the government
guarantees the obligations of various financial institutions. Panics may still occur if
the government is perceived as unwilling or unable to live up to those guarantees.
12.
To be considered money the currencies would have to fulfill the functions of money.
They are only partially fulfill it since they only have limited acceptability as a
Page 748, Answers to End-of-Chapter Questions
medium of exchange, store of value, and unit of account. Thus, while they are partial
monies, we would not consider them full monies.
Problems and Exercises
1. a. If individuals hold no cash, the simple money multiplier is the reciprocal of the
reserve requirement. Thus for the following reserve requirements the simple
multiplier is found by dividing the requirement percentage into 1: 5%, 20; 10%, 10;
20%, 5; 25%, 4; 50%, 2; 75%, 1.33; 100%, 1.
b. If individuals hold 20% of their money in the form of cash, the multiplier becomes
(1)/(r + c) and so for the following reserve ratios their multipliers are now: 5%, 4.0;
10%, 3.33; 20%, 2.5; 25%, 2.22; 50%, 1.43; 75%, 1.05; 100%, 0.83.
2.
a.
b.
c.
d.
For a deposit of $100 and a reserve ratio of 5%,
the bank can lend out $95.
There is now $195 of money.
The multiplier is 20.
John's $100 will ultimately turn into $2000
3. a. Neither
b. Both
c. M2
d. Both e. Neither f. Neither
g. Both
4. a. money; b. not money; c. not money; d. money; e. money;
f. not money;
g. not money
5. a.
b.
c.
d.
Nothing happens to M2. M1 declines by $200
Nothing happens to M1 or M2
Nothing happens to M2, M1 rises by $50
Nothing happens to M1 or M2.
Web Questions
1. a. Beenz is an alternative to money that aims to become globally acceptable. Its main
purpose is to reward on-line consumer behavior. You can earn beenz by signing up,
and ‘surfing’ various specific websites. Likewise, one can spend beenz at various
websites to purchase goods etc.
b. Currently, beenz is not as liquid as money. It will take time before it can be used as a
generally acceptable medium of exchange. However, it does fulfill the three functions
of money as far as the websites where beenz is accepted are concerned.
c. Beenz is money in the sense that it has the potential to fulfill the functions of money
if people start believing that others will accept it in exchange for their goods.
2. a. There is about $500 billion of U.S. currency in circulation today but most of it resides
outside of the U.S. Assuming that the world population is about 6 billion, this means
that there is approximately $83 per person in the world.
b. People typically withdraw cash at ATMs over the weekend, so there is more cash in
Page 749, Answers to End-of-Chapter Questions
circulation on Monday than on Friday.
c. 1.5 years.
d. Most of this is in the form of U.S. Government Securities owned by the Federal
Reserve System. Some of it also consists of gold certificates, special drawing rights,
and “eligible” paper such as bills of exchange or promissory notes.
e. Bureau of Engraving and Printing.
Appendix A
1.
Students gain a financial asset and the government incurs a financial liability.
2.
It is a financial asset because it has value due to an offsetting liability of the Federal
Reserve Bank.
3.
No. In economic terminology he is saving. Investing is the act of spending the money
on real investment goods in economic terminology.
4.
No, she is not correct. While a loan is a loan, that loan is a financial asset to the one
issuing the loan because it has value just as does a bond.
5.
An investment bank facilitates borrowing. It does not take in deposits and often does
not make loans. A commercial bank takes in deposits and makes loans.
6.
False. The difference has nothing to do with which is more important. The difference
is that in primary markets new issues are sold and in secondary markets existing
stocks and bonds are resold. The secondary market is much larger.
7.
The prospects must not be very good or the interest rate must be extremely high.
Generally, stocks sell for a minimum of multiples of ten or twelve times earnings.
This multiple can be roughly determined by dividing the expected earnings (the
annuity) by the interest rate. More recently the average price/earnings ratio has been
30, a historically high figure.
8.
On the surface it might seem that the primary market contributes more to the
production of tangible real assets because it is here that new firms find the financing
for their ideas. A secondary financial market only transfers existing financial assets
from one saver to another, and such a transaction does not represent any new savings.
However, the existence of a secondary market plays an important role in enticing
people to buy in the primary market because it makes their financial asset more
liquid, and, hence, more valuable. Thus on a deeper level it is hard to say which
contributes more.
9.
Money market assets usually pay lower interest rates than do longer-term capital
assets because they offer the buyer more liquidity and less risk of asset value
fluctuation.
Page 750, Answers to End-of-Chapter Questions
10.
Asset
Liability
Market
a. Lamar
Credit Union
money
b. Pension USA Sandra
capital
First bank initially held the asset, but then FNMA held it. Finally, Pension USA held
it as owner of share in FNMA mortgage fund. All along it was a liability to Sandra.
Asset
Liability
Market
c. First Bank
Sean
money
d. residents of
Providence
Providence
capital
e. Lanier
the firm
capital
No financial asset has been created, it has just changed hands.
Asset
f. Lanier
Liability
the firm
Market
capital
11. a. Technically, a rise in stock prices does not imply a richer economy. If, however, the
rise in stock prices reflects underlying real economic improvement such as finding
the cure for cancer or a technological advance, society will be richer not because of
the rise in stock prices, but because of the underlying cause of their rise.
b. We disagree with this statement. If both the real and financial asset are worth
$1,000,000, then they have the same value as long as they are valued at market
prices. Just as financial assets bear a risk of no repayment, real assets bear a risk of a
fluctuation in prices.
c. Although financial assets have a corresponding liability, they facilitate trades that
could not otherwise have taken place and thus have enormous value to society.
d. This is false. The value of an asset depends not only on the quantity but also its price
per unit. The price of land per acre in Japan exceeds that in the U.S. by so much that
its total value also exceeds that in the U.S.
e. This is false. The stock market valuation depends on the supply and demand for
existing stock. There is, however, a relationship between relative growth in GDP and
the rise in stock prices to the extent that growth in stock prices and GDP growth both
reflect economic well-being in a country. Also, many of the companies are
multinational companies and where the company is based may not reflect where its
value added is generated.
Appendix B
1. Starting with the initial balance sheet:
Assets
Cash
Loans
Phys assets
20,000
225,000
105,000
Liabilities & Net Worth
Demand dep 200,000
Net worth 150,000
Page 751, Answers to End-of-Chapter Questions
Total
350,000
Total 350,000
a. If an immigrant enters the country and deposits $10,000 in a bank the result is:
Assets
Cash
Loans
Phys assets
Total
30,000
225,000
105,000
360,000
Liabilities & Net Worth
Demand dep
210,000
Net worth
150,000
Total
360,000
b. If the bank keeps 10% of this deposit and lends the rest, the result is:
Assets
Liabilities & Net Worth
Cash
21,000
Demand dep
210,000
Loans
234,000
Net worth
150,000
Phys assets
105,000
Total
360,000
Total
360,000
c. 80% of the loan amount ($9000) gets deposited back in the bank, and the result is:
Assets
Liabilities & Net Worth
Cash
28,200
Demand dep
217,200
Loans
234,000
Net worth
150,000
Phys assets
105,000
Total
367,200
Total
367,200
d. After the money multiplier is all through multiplying, the initial $10,000 will have
become $40,000 (if everyone deposits 80% of what they receive back into the bank),
and the final balance sheet will be:
2. a.
Assets
Cash
Loans
Phys. assets
Total Assets
10,000
- 1,000
9,000
100,000
50,000
159,000
Liabilities
Demand deposits
Net worth
50,000
- 1,000
49,000
110,000
Total Liabilities and
net worth
159,000
b. The reserve ratio is now 18 percent. This is less than the required 20 percent. The
bank must decrease loans by $800 to meet the reserve requirement. But this shows up
as $800 less in demand deposits and 800 less in cash. The bank must again reduce
loans but this time by 640. Demand deposits once again decline. This continues until
the final position indicated by the following T-account:
Assets
Cash
9,000
Liabilities
Demand deposits
Page 752, Answers to End-of-Chapter Questions
45,000
Loans
Phys. assets
Total Assets
96,000
50,000
155,000
Net worth
110,000
Total Liabilities and net worth155,000
c. The money multiplier is 5.
d. Total money supply declined by 5000.
3. a. The bank is holding $7500 in excess reserves.
b. $50,000 new money would be created: (1/0.15) * 7,500.The final T-account is shown
below:
Assets
Cash
Loans
Phys. assets
Total Assets
30,000
370,000
350,000
750,000
Liabilities
Demand deposits
Net worth
200,000
550,000
Total Liabilities and net worth
750,000
Chapter 30: Monetary Policy and the Debate about Macro Policy
Questions for Thought and Review
1.
The Fed is a semi-autonomous agency of the federal government. Although it is
owned by member banks, its officials are appointed by government. It is a creation of
Congress, but has much more independence than do most public agencies.
2.
There are few regional Fed banks in the western part of the U.S. because in 1913,
when the Fed was established, the West and South were smaller and less important
economically than the rest of the country, so fewer banks were established there. As
these regions grew, the original structure remained because no one wanted to go
through the political wrangling that restructuring would bring about.
3.
Six explicit functions of the Fed include: 1) conducting monetary policy; 2)
supervising financial institutions; 3) serving as a lender of last resort; 4) providing
banking services to the U.S. government; 5) issuing coin and currency; and 6)
providing financial services to commercial banks.
4.
Three tools by which the Fed can affect the money supply are: 1) changing the
reserve requirement; this changes the amount of reserves banks need and thereby
changes the money supply; 2) changing the discount rate; this changes the cost of
borrowing by banks from the Fed and thereby changes the money supply (Actually, it
works more as a signal.); and 3) open market operations; as the Fed buys and sells
bonds, it change reserves and thereby changes the money supply.
Page 753, Answers to End-of-Chapter Questions
5.
When the Fed buys bonds the price of bonds rises and the interest rate falls.
6.
The Fed funds rate is the interest rate that banks change one another for Fed Funds or
reserves. As the Fed buys and sells bonds, it changes reserves and thereby directly
affects this short term overnight interest rate. Other longer-term interest rates such as
the Treasury Bill rate are only indirectly affected.
7.
Defensive policies are simply changes to offset fluctuations in the demand for money.
Therefore a change in the direction of monetary policy would be an offensive action.
8.
When the Fed takes money out of the economy, banks are in violation of Fed
regulations and have no choice but to contract their loans in order to meet their
reserve requirements. When the Fed puts money into the economy, banks have excess
reserves but there is no regulation that they are violating. Although they may have a
financial incentive to make loans, they are not required to do so. Since they are not
required to make loans, the phrase, "You can lead a horse to water, but you can't
make it drink," is relevant.
9.
To increase income by 240, investment should increase by 80 (the income multiplier
is 3). To increase investment by 80 requires decreasing the interest rate by 4
percentage points (investment increases by 20 for every 1 percentage point drop in
interest). To change the interest rate by 4 requires a change of 20 in the money supply
(each change of 5 in the money supply changes the interest rate by 1 percent). Since
the money multiplier is 4, reserves should increase by 5. Thus the recommended
policy is an open market purchase that would increase reserves by 5.
10.
The nominal interest rate is equal to the real interest rate plus the expected inflation
rate. If the nominal interest rate is 6 percent and the expected inflation rate is 5
percent, the real interest rate is 1 percent.
11.
If we consider the example of an open market sale by the Fed, the initial transaction
or "splash" would be that a person writes a check to the Fed, and the Fed presents it
to the person's bank for payment. The bank now must adjust to this change, and the
"ripples" will show up on its balance sheet. The loss of cash to the Fed means that the
bank's reserves are too low, and the bank must figure out a way to meet its reserve
requirement. It may call in loans to do so, but that in turn could mean that someone
paid the loan from a checking account, which has further balance sheet implications.
12.
Treasury bills pay interest; cash does not.
13.
From 1996 to 1997 the Fed tightened monetary policy slightly; in 1998 it loosened it
slightly and in 1999 it tightened it.
14.
The Taylor Rule suggests that the Fed will increase the Fed Funds rate by 1.5%
Page 754, Answers to End-of-Chapter Questions
because inflation is 1% above desired.
Problems and Exercises
1.
In each of the three cases, expansionary monetary policy shifts the AD curve to the
right. The difference is whether the shift results in an increase in real output, an
increase in the price level, or a combination of the two.
a. In the short run, when the economy is in the Keynesian range, the shift of AD curve
to the right from AD0 to AD1 will increase output with no change in the price level.
The economy moves from point A to point B (output rises to Q1 but the price level
remains at P0) in the graph below.
b. When the economy is in the intermediate range, the effect if the shift of AD curve to
the right depends upon how close to potential the economy is. The closre to
potential, the more the increase will result in an increase in the price level and not an
increase in real output. One example if for the economy to move from point B to C
shown in the graph below. The AD curve shifts from AD1 to AD2. Input prices
begin to rise and the AS curve shifts from AS0 to AS1. Output rises to Q2 and the
price level rises to P1.
c. When the economy is in the Classical range, the effect of the shift of the AD curve to
the right is entirely on the price level, not output. Graphically this can be shown as
an economy moving from point C to point D in the graph below. The AD curve shifts
from AD2 to AD3, but the AS curve shifts up so much that output remains at Q2
while the price level rises to P2.
Potential
Output
Price
Le ve l
D
P2
C
P1
P0
AS2
A
AS1
AD3
B
AS0
AD2
AD1
AD0
Q0
Q1 Q2
Re al Output
2. a
If people hold no cash, the money multiplier is (1/r). If this is equal to 3, then the
current reserve requirement is 33%. To increase the money supply by 200 the Fed
should lower the reserve requirement to 32%.
b. Lowering the discount rate will encourage banks to borrow. This will increase the
amount of reserves in the system so the money supply increases. If the Fed wishes to
increase the money supply by 200, and the multiplier is 3, reserves must be increased
by 66.67. If banks will borrow an additional 20 for every point the discount rate is
lowered, the Fed should lower the rate by 3.33 percentage points.
Page 755, Answers to End-of-Chapter Questions
c. To increase the money supply by using open market operations the Fed should buy
bonds, thus increasing the level of reserves in the banking system. To achieve an
increase of 200 (if the multiplier is 3) the Fed should buy 66.67 worth of bonds.
3. a. Decreasing the reserve requirement from 20 percent will provide banks with excess
reserves and will increase the multiplier. To calculate exactly how much we would
need to know the current money supply.
b. The Fed would buy $400,000 worth of bonds, increasing reserves, and so increase the
money supply.
4. a. Increasing the reserve requirement would lower the multiplier, calculated as [l/(r +
c)]. To calculate exactly how much we would need to know the current money
supply.
b. The money multiplier is [l/(r + c)] = 2.5. If the Fed sold $800,000 worth of bonds it
would decrease reserves and so decrease the money supply.
c. This part of the questions requires students to obtain information from a local bank,
and reevaluate a and b in view of what they learn.
5. a. The money multiplier, assuming no cash holding, would be one.
b. The money supply would decrease enormously.
c. This could be offset by the Federal government buying up Treasury bills, directly
increasing the money supply or by the Federal government making loans directly.
d. Since this eliminates a significant role of banks as lenders, eliminating a market
within which they receive at least a normal profit they would most likely oppose it.
6. a. This would increase excess reserves enormously.
b. Banks would most likely favor this proposal because they would now earn interest on
their assets held at the Fed.
c. Central banks would likely oppose this because it would reduce their superiority and
may require that they ask Congress for appropriation to pay the interest reducing their
political independence.
d. This would increase the interest rate paid by banks because the additional interest
would increase their profit margin. The initial increased profit margin would shift the
demand for depositors out as new banks entered the market and as existing banks
competed for more deposits. This would increase the interest paid to depositor until
the normal profits are once again earned.
7.
For every percentage point inflation is above (below) the Fed’s inflation target, the
Fed funds rate will rise (fall) by 1.5 percentage points; for every percentage point the
economy’s total output is above (below) its potential output, the Fed funds rate will
rise (fall) by half a percentage point.
a. Fed funds rate will fall 2.5 percentage points.
b. Fed funds rate will rise 4.5 percentage points.
c. Fed funds rate will rise 0.5 percentage point.
Page 756, Answers to End-of-Chapter Questions
Web Questions
1. a. Alan Greenspan is chairman. He has been the chairman since August 11, 1987.
b. The seven members are nominated by the President and confirmed by the Senate. The
current members others than Greenspan are Roger W.Ferguson Jr, Edward W.Kelley
Jr, Laurence H.Meyer and Edward M.Gramlich. The remaining two seats are
currently vacant.
c. All of them have an economics bachelor’s degree, a Ph.D. in economics and/or a
MBA degree. Some of them have held posts in private consulting firms, and most of
the have had prior experience in public economics related commissions and
committees. Some like Meyer are specialists in economic forecasting.
2. a. According to the report of February 2000, the economy has continued to perform
exceptionally well. Inflation has remained low due to the increases in productivity.
b. Inflation.
c. In order to sustain growth with low levels of inflation, the Fed has been encouraging
tighter monetary policy. Proof of this is the recent half percentage-point increase in
the interest rate.
Appendix A
1.
Since the question does not indicate a reserve requirement or an initial bank balance
sheet, let’s assume a 10 percent reserve requirement and an initial bank balance sheet
shown below.
Assets
Reserves
T-bill holdings
Loans
Total Assets
Initial Bank Balance Sheet
Liabilities
100,000,000
Demand deposits
0
Net worth
905,000,000
1,005,000,000
Total Liabilities
1,000,000,000
5,000,000
1,005,000,000
First, individuals buy $1 million T-bills from the Fed, by withdrawing $1,000,000
from the bank leaving the bank with no reserves.
Assets
Reserves
T-bill holdings
Loans
Total Assets
99,000,000
0
905,000,000
1,004,000,000
Liabilities
Demand deposits
Net worth
Total Liabilities
999,000,000
5,000,000
1,004,000,000
To meet its reserve requirement the bank must call in $900,000 in loans. These loans
are repaid by individual withdrawals of demand deposits of $900,000.
Assets
Liabilities
Page 757, Answers to End-of-Chapter Questions
Reserves
99,000,000
loans repaid
+900,000
deposits withdrawn -900,000
Demand deposits
T-bill holdings
Loans
Total Assets
Net worth
0
904,000,000
1,003,000,000
Total Liabilities
998,000,000
5,000,000
1,003,000,000
Again the bank calls in loans to meet reserve requirements. Each round, the amount
called in gets smaller and smaller until the bank arrives at its final position with money
supply having fallen by 10,000,000.
Liabilities
Reserves
T-bill holdings
Loans
Total Assets
2.
90,000,000
0
905,000,000
995,000,000
Assets
Demand deposits 900,000,000
Net worth
5,000,000
Total Liabilities
995,000,000
Since the question does not indicate a reserve requirement or an initial bank balance
sheet, let’s assume a 10 percent reserve requirement and an initial bank balance sheet
shown below.
Assets
Reserves
T-bill holdings
Loans
Total Assets
Initial Bank Balance Sheet
Liabilities
100,000,000
Demand deposits
0
Net worth
905,000,000
1,005,000,000
Total Liabilities
1,000,000,000
5,000,000
1,005,000,000
First, individuals sell $2 million T-bills to the Fed, and deposit the $2 million in the
bank. The bank now has more reserves than is required
Assets
Reserves
T-bill holdings
Loans
Total Assets
102,000,000
0
905,000,000
3,005,000,000
Liabilities
Demand deposits
Net worth
1,002,000,000
5,000,000
Total Liabilities
3,005,000,000
It has excess reserves of $1.8 million, which it lends out. These loans are redeposited
at the bank as demand deposits
Assets
Reserves
loans given
new deposits
102,000,000
-1,800,000
+1,800,000
Liabilities
Demand deposits
New deposits
1,002,000,000
+1,800,000
Page 758, Answers to End-of-Chapter Questions
T-bill holdings
Loans
Total Assets
0
906,800,000
1,008,800,000
Net worth
Total Liabilities
5,000,000
1,008,800,000
It still has excess reserves of 1.62 million, which it lends out. Each round, the amount
called in gets smaller and smaller until the bank arrives at its final position with money
supply having risen by 20,000,000.
Liabilities
Reserves
T-bill holdings
Loans
Total Assets
102,000,000
0
923,000,000
1,025,000,000
Assets
Demand deposits 1,020,000,000
Net worth
5,000,000
Total Liabilities 1,025,000,000
Chapter 31: Inflation and Its Relationship to Unemployment and Growth
Questions for Thought and Review
1.
Lenders lose out because they are paid a fixed nominal interest rate. They would not
lose out if the interest rate were indexed to inflation or if the lender was able to adjust
the interest rate periodically.
2.
Adaptive expectations.
3.
2 percent.
4.
MV = PQ.
5.
The three assumptions are that velocity is constant, real income is constant and the
direction of causation is from money to prices.
6.
Inflation is always and everywhere a monetary phenomenon.
7.
Inflation will be 10 percent.
8.
Financial institutions changed enormously and financial markets become increasingly
connected internationally, increasing the flow of money among countries.
Page 759, Answers to End-of-Chapter Questions
9.
Developing countries. In those countries the money supply is growing so quickly it
dominates any other factors that might affect the relationship.
10.
Governments and central banks sometimes increase money supply even when they
know the consequences because sometimes the political ramifications of not
increasing the money supply (which can include a collapse of government) are
thought to be worse.
11.
An inflation tax is an implicit tax on the holders of cash and the holders of any
obligations specified in nominal terms. The holders of cash and the holders of any
obligations specified in nominal terms pay the tax.
12.
Quantity theorists are more likely to support rules because they have less trust in
government undertaking beneficial actions. Rules limit those actions.
13.
From right to left.
14.
The labor market is central to inflation because wages make up 60% of the costs of
production. These labor markets are often influenced by social factors and only
slowly respond to economic pressures.
15.
The insider/outsider theory of inflation divides workers into insiders and outsiders. It
is an example of an institutionalist theory of inflation where social pressures prevent
economic pressures from working. In it insiders push up wages, and the outsiders
find themselves experiencing the unemployment. So there is little pressure on
insiders not to raise wages.
16.
The quantity theory emphasizes control of the money supply; Institutionalist theories
also require control of the money supply, but they have other methods such as labor
market structural changes or income policies that are used in conjunction with control
of the money supply.
17.
The short-run Phillips curve is illustrated on page 714 of the text in Figure 31-3(a).
The short-run curve shows the trade-off between inflation and unemployment when
expectations of inflation are constant. The long run Phillips curve is shown in Figure
31-4(b) as the vertical curve. The long-run curve shows the trade-off (or lack thereof)
when expectations of inflation equal actual inflation.
18.
No, as long as expectations of inflation are constant, the economy will stay on the
same short run Phillips curve.
19.
It depends. With short-run, long-run, and shifting curves, just about any combination
of inflation and unemployment rates can fit some Phillips curve. So, yes, the Phillips
curve is a figment of economists' imaginations. But then again, aren't all models
simply structures imposed on reality and doesn't reality only get interpreted through
Page 760, Answers to End-of-Chapter Questions
imaginary imposed structures? If so, to suggest that the Phillips curve is "nothing but
a figment" is incorrect. Reality is itself a figment of imagination. (If you follow this
answer, you might consider shifting to a philosophy major.)
20.
Economists see a tradeoff between inflation and growth because low inflation
reduces price uncertainty and thereby encourages investment, increasing the
efficiency of the market system.
Problems and Exercises
1. a.
b.
c.
d.
Use the equation, MV = PQ. Real output is $2,000.
Nominal output is $4,000.
If the money supply rises from 500 to 600, the price level will rise from 2 to 2.4.
If the government established price controls, either shortages would result if the
economy were perfectly competitive, or real output would rise if the economy had
monopolistic elements.
e. We would look at the empirical evidence, and see whether it has remained constant
in the past when similar circumstances have prevailed.
Inflation
2. a. The economy is at point A on short-run and long-run Phillips curve on the graph
below.
b. The answer to this question really
LP
hinges on what kind of change would
be popular. Should you try to cut
B
C
unemployment further? If so, then we
would
recommend
increasing
A
SP (Pe > 5%)
government expenditures, moving the 5%
SP (Pe = 5%)
economy to a point such as B in the
graph on the right. Or would a better
5%
strategy be cutting inflation? If so,
Une mployme nt rate
then we would recommend reducing
government expenditures, which will
increase unemployment while reducing inflation.
c. We have chosen the “lower unemployment” option. An increase in aggregate
expenditures will cause a movement up along the short run Phillips curve.
Unemployment will fall, but inflation will rise. In the long run, as expectations of
inflation adjust to actual inflation, the short run Phillips curve shifts up.
Unemployment returns to its target rate of 5%, but inflation is higher than before as
shown as point C on the graph on the right.
3. a. Stopping inflation tends to transfer money from debtors to creditors. Creditors are
generally rich, and can golf regardless of their wealth. Debtors, faced with a decrease
in their wealth, must cut back on discretionary expenditures, of which golf is one.
b. Since the exchange rate was fixed, any differential in inflation rates between the two
countries could not be offset a change in the exchange rate. The fact that goods in
Page 761, Answers to End-of-Chapter Questions
dollar equivalent pesos in Argentina were higher than in NYC suggests that the
Argentinean inflation rate remained greater than in the U.S. and the high prices of
goods were serving as an anchor on the economy.
c. In an inflation (with interest rates falling behind inflation), people look for real assets
to buy to protect their wealth. This increases the demand for goods relative to
services increasing their price. When the inflation is stopped, the opposite occurs.
d. One reason why luxury auto dealers were shutting down was the same as the
argument given in (a). A second reason is equivalent to that given in (c). A third
reason is that wealthy Argentineans who would most likely purchase such a car also
probably had foreign bank accounts denominated in dollars. The car in dollars was
cheaper because the peso was overvalued at the fixed exchange rate. The demand for
luxury cars fell as Argentineans substituted dollar-denominated luxury cars for pesodenominated cars.
4. a. The advantage of indexing grades is that it provides a benchmark with which to
measure a student's performance in his/her class. It would distinguish between an A
received in a difficult class in which many did not receive A's and an A earned in an
easy class in which A's were plenty. The disadvantage is that it would not distinguish
between an A earned among A's in a class where A's were given generously without
work an A earned among A's in a class of geniuses. It might result in professors
making distinctions among bright students whose abilities are virtually the same just
to make a given distribution of grades.
b. This is parallel to the argument for fixed exchange rates that place limitations on
individual grade policies (their distributions) for professors.
5. a. One would expect real output to decline.
b. One would expect unemployment to rise.
c. One would expect inflation to fall.
6. a. He would likely be a quantity theorist since they see inflation most connected to long
term growth because low inflation means that the informational job of prices is
working better and more investment will take place.
b. Inflation can affect household decisions in a number of ways. It can add uncertainty
about the future, leading them to save less. Alternatively, it could lead them to
temporarily supply more labor than they would otherwise, causing a temporary spurt
in growth, and then a fall in growth once they recognize their mistakes.
7. a. Answers may differ. Five goods we buy frequently are newspapers, soda, gas, shirts,
and coffee.
b. This requires research by the student. Answers will depend on goods chosen.
c. Answer will depend on good chosen.
Web Questions
1. a.
Page 762, Answers to End-of-Chapter Questions
Year
1959
1960
1961
1962
1963
1964
1965
1966
1967
1968
1969
1970
1971
1972
1973
1974
1975
1976
1977
1978
1979
1980
1981
1982
1983
1984
1985
1986
1987
1988
1989
1990
1991
1992
1993
1994
1995
1996
1997
1998
1999
M1
billions
140.3767
140.3108
143.0525
146.4758
150.9275
156.7958
163.465
170.9617
177.6692
190.1333
201.4192
209.1208
223.1692
239.0542
256.3458
269.2033
281.4358
297.1942
319.9783
346.305
372.7117
395.7467
424.8842
452.9992
503.165
538.6517
586.9975
666.5058
743.6267
774.9767
782.4733
811.0733
859.5342
966.3583
1078.999
1145.484
1143.095
1106.316
1069.881
1080.712
1102.438
GNP
billions
507.425
527.375
545.625
586.525
618.675
664.375
720.1
789.3
834.075
911.45
985.35
1039.675
1128.6
1240.425
1385.55
1501
1635.175
1823.925
2031.4
2295.875
2566.375
2795.55
3131.35
3259.225
3534.95
3932.75
4213
4452.85
4742.475
5108.325
5489.05
5803.25
5986.225
6318.95
6642.325
7054.3
7400.55
7813.175
8300.725
8759.95
9256.15
Velocity
3.614739
3.758619
3.814159
4.004244
4.099154
4.237198
4.405224
4.616824
4.69454
4.793741
4.892037
4.971647
5.05715
5.188887
5.405003
5.575711
5.810117
6.137149
6.348555
6.629633
6.885685
7.063989
7.369891
7.19477
7.025429
7.301101
7.177203
6.680887
6.377495
6.591586
7.014999
7.155025
6.964499
6.538931
6.156006
6.158357
6.474134
7.062337
7.758551
8.105724
8.396077
b. One dollars in 1960 supported approximately $3.8 in total income. This rose to about
Page 763, Answers to End-of-Chapter Questions
$5.0 in 1970, and $7.1 in 1980. By 1990, one dollar circulated enough to support
approximately $7.2 in total income.
c. As can be seen the velocity of money has on average been rising over the last couple
of decades.
d. The quantity theory of money assumes that the velocity of money is constant. This is
obviously not the case as can be seen by the above chart. Thus, this theory will not be
able to predict accurately the growth in nominal GDP due to the growth of the money
supply.
2. a. The article suggests that there is an inverse relationship between the two (i.e. as
interest rates rise, inflation falls).
b. The article emphasizes the quantity theory viewpoint since it says that changes in
interest rates (due to changes in money supply) will lead to changes in the inflation
rate.
c. The Fed has to estimate the state of the economy and then make decisions regarding
interest rates.
d. The terrain could include variables like production, employment, consumer
expenditures and their expectations etc.
Chapter 32: Open Economy Macro: Exchange Rate and Trade Policy
Questions for Thought and Review
1.
If a country is running a balance of trade deficit, the amount of goods it is exporting
is less than the amount of goods it is importing. This is only one part of the current
account, which is the part of the balance of payments that lists all short-term flows of
payments. A deficit in merchandise could be offset by a surplus in other areas of the
account.
2.
When someone sends 100 British pounds to a friend in the United States, the
transaction will show up in the component of the current account called net transfers,
which include foreign aid, gifts, and other payments to individuals not exchanged for
goods or services. It will also appear on the capital account as a receipt of foreign
currency just like the purchase of a British stock or bond.
3.
The capital account measures the flow of payments between countries for assets such
as stocks, bonds, and real estate, and a surplus means that capital inflows were more
than capital outflows. To buy United States assets, foreigners need dollars, so the net
Page 764, Answers to End-of-Chapter Questions
capital inflows represent a demand for dollars that can balance the excess supply of
dollars due to a current account deficit.
In the short run, a current account deficit balanced by a capital account surplus is nice
because current expenditures, which include the trade balance, give society
immediate pleasure.
4.
In the reverse situation, a capital account deficit will mean an excess supply of
dollars, which could be offset by the added demand for dollars created by a current
account surplus. In the long run, capital account deficits are nice because you are
building up holdings of foreign assets.
5.
A non-convertible currency cannot be freely exchanged with other currencies. Such a
currency can only be bought from the country's government and sold to it. The
exchange rates of such currencies do not fluctuate in response to shifts in supply and
demand. The major disadvantage of this is that it makes trade difficult. Yet countries
use this to avoid making the economic adjustments that international considerations
would otherwise force upon them.
6.
Holding the exchange rate above the equilibrium market exchange rate will make a
country's exports more expensive and its imports cheaper than they otherwise would
have been. It will also require the country to finance the deficit using reserves or
borrowing to do so. It can allow the government to temporarily not make the
contractionary macro adjustments that otherwise would be necessary to bring the
economy into equilibrium.
7.
We would use a combination of purchasing power parity, current exchange rates, and
estimates of foreign exchange traders to determine the long-run exchange rate of the
Neverback. This combination approach can only be justified by the "that's all we have
to go on." defense. Since no one really knows what the long run equilibrium
exchange rate is, and since that exchange rate can be significantly influenced by other
countries policies, the result we arrive at could well be wrong.
8.
Both fixed and flexible exchange rate systems have advantages and disadvantages.
While fixed exchange rates provide international monetary stability and force
governments to make adjustments to meet their international problems, they have
some disadvantages as well: they can become unfixed, creating enormous instability;
and their effect of forcing governments to make adjustments to meet their
international problems can be a disadvantage as well as an advantage. Flexible rates
provide for orderly incremental adjustment of exchange rates and allow governments
to be flexible in conducting domestic monetary and fiscal policies, but also allow
speculation to cause large jumps in exchange rates (and, as before, the government
flexibility may be a disadvantage too). Given the pluses and minuses of both systems,
most policymakers have opted for a policy in between-partially flexible exchange
Page 765, Answers to End-of-Chapter Questions
rates.
9.
He will more likely prefer fixed exchange rates. They provide an anchor, which
restricts government temptations to use expansionary monetary policy.
10.
They will sell that currency, which will force the government to use reserves to
protect the currency.
11.
Three types of regulatory restrictions are tariffs, quotas and non-tariff barriers. Tariffs
are taxes on imports. Quotas are quantity restrictions on imports and non-tariff
barriers are a combination of regulatory restrictions that make importing harder. The
advantage of tariffs is that they bring in revenue. The advantage of quotas is that
politically, they are often more easily implemented than tariffs. The advantage of
non-tariff barriers is that governments can claim that they are not really barriers to
trade.
12.
A tariff is a tax on imports; a quota is a quantitative restriction on imports.
13.
As you can see in the graph on the right, a tariff of
T brings in revenue to the government shown by
the boxed area while a quota of Q1 that achieves
the same quantitative restriction gives that revenue
to producers in the form of increased profits.
ST
Price
S0
PT
P0
D
Q1 Q0
Quantity
14.
An inefficient customs agency can act as a non-tariff barrier. It restricts imports in the
same way that a tariff or quota does.
15.
No. Assuming that we had suggested free trade, we would continue to suggest free
trade. The recession will likely reduce imports, improving the trade balance, but it
will also increase unemployment, which will increase political pressure to restrict
imports. Since we have been convinced by economic arguments for free trade, we
would not change our advice.
16.
He was advocating significant trade restrictions. These trade restrictions would have
likely provoked retaliation by our trading partners, hurting international cooperation,
and hurting the world economy.
17.
No. It is extremely difficult to affect exchange rates. Since we don't know what the
correct exchange rates are, it is probably best not to try to significantly change the
exchange rates determined by the market by foreign exchange intervention. If one is
Page 766, Answers to End-of-Chapter Questions
going to change exchange rates one must change one's domestic monetary and fiscal
policies.
18.
The U.S. would want to hold up the value of the dollar to help prevent the surge in
prices that would result from the fall in exchange rates, and to keep foreigners from
buying our assets cheaply. Other countries would want a higher value of the dollar in
order to keep their goods competitive with U.S. goods.
Problems and Exercise
a.
b.
c.
d.
The graph to the right shows the fundamental
S0
analysis of the supply and demand for British
S1
pounds sterling in terms of dollars, and the
P1
effect of the following changes:
A rise in the UK price level causes foreign
P0
goods to become cheaper. British demand for
D1
P2
foreign currencies will tend to increase, and
D0
foreign demand for pounds will tend to
decrease. Thus supply of pounds shifts
Q0 Q1 Q2
outward fro S0 to S1 and the demand for the
Quantity of pounds
pound shifts inward from D1 to D0. The
exchange rate value of the pound falls from P1 to P2.
The United States price level rising has the effect noted above, but the graph
illustrates the impact of the change in the value of the dollar on the pound in terms of
dollars. As the value of the dollar falls, the value of the pound in terms of the dollar
rises, and so shifts are opposite to those of part a. That is the supply of pounds shifts
inward fro S1 to S0 and the demand for the pound shifts outward from D0 to D1. The
exchange rate value of the pound rises from P2 to P1.
A boom in the UK economy means an increase in its income, causing an increased
demand for imports and an increase in the demand for the foreign currency to buy
those imports, thus resulting in an increase in the supply of pounds. (This may also
set off an expectations effect.) Thus, the supply of pounds shifts outward from S0 to
S1. If demand is at D0, the exchange rate value of the pound falls from P0 to P2.
If interest rates in the UK rise, there will be an increased demand for its assets, so the
demand for pounds will increase from D0 to D1 and the supply of pounds will
decrease from S1 to S0 as fewer British investors sell their pounds to buy foreign
assets. The exchange rate value of the pound rises from P2 to P1.
Price of pounds in
dollars per pound
1.
Page 767, Answers to End-of-Chapter Questions
Price of Neverbacks in
dollars per Neverback
2. a. This is an enormous change. In order to
S0
bring it about, the Never-Never
S1
government would have to run an
$20
enormously expansionary monetary
policy, reducing the real interest rate
possibly to negative amounts, and
$10
D0
probably generating significant inflation.
D1
b. Holders of Neverbacks will demand
Quantity of Neve rbacks
foreign currencies (increase supply of
Neverbacks) since the return on
Neverback assets has declined. This is shown as a rightward shift in the supply of
Neverbacks. Likewise, potential foreign investors will demand fewer Neverbacks for
the same reason. This is shown as a leftward shift in the demand for Neverbacks from
D0 to D1. The effect is to reduce the exchange rate value of the Neverback to $10 per
Neverback.
3. a. Supplier b. Supplier c. Supplier
d. Demander e. Demander f. Demander.
4. a. We would suggest buying U.S. dollars and selling currencies of EU.
b. We would suggest buying U.S. dollars since interest rates in U.S. are expected to be
higher, the quantity of U.S. assets demanded will rise, and thus the demand for
dollars will increase.
c. Since the market will likely already have responded to the higher expected interest
rates, the rise will likely have the same effect as a fall in interest rates. Thus I would
suggest selling U.S. dollars.
d. We suggest selling U.S. dollars by similar reasoning in b.
e. We would suggest selling U.S. dollars in the expectations of a decrease in demand
for U.S. dollars as U.S. goods become more expensive. Also U.S. denominated assets
such as bonds will be worth less with greater inflation making foreign assets more
attractive to investors.
5. a. Current account b. Current account
d. Current account e. Capital account
c. Current and capital accounts
6. a. Three assumptions of the law of one price are (1) zero transportation costs; (2) the
goods are tradable and (3) there are no barriers to trade. (There are many others.)
b. For it to apply directly to labor would have to be completely mobile. Thus it does not
apply directly. However, assuming capital is flexible, there will be significant indirect
pressure toward an equalization of wage rates.
c. Since capital is more mobile than labor, we would expect that the law of one price
would hold more for capital than for labor.
Web Questions
1. a. Country
Implied PPP of the Dollar
Exchange Rate
Page 768, Answers to End-of-Chapter Questions
Australia
Britain
Israel
Malaysia
Russia
1.03
1.32
5.78
1.8
15.7
1.68
1.58
4.05
3.8
28.5
All the currencies are overvalued except the Isreali shekel, which is undervalued.
b. The Big Mac Index is limited because it measures purchasing power parity only in
terms of Big Macs. People don’t just consume Big Macs. Including more goods in a
purchasing power parity index would make the index more accurate.
c. In designing an index it would be important to include a wide variety of goods in
order to measure overall changes in purchasing power parity, everything from food
and clothing to tires and light bulbs.
d. You could check the validity of the Big Mac index by comparing its implied
purchasing power parities in U.S. dollars to actual exchange rates.
2. a. The mission of the International Trade Administration is to encourage, assist, and
advocate U.S. exports, to ensure that U.S. business has equal access to foreign
markets, and to enable U.S. businesses to compete against unfairly traded imports
and to safeguard jobs and the competitive strength of American industry.
b. Two hot issues on the ITA website include food biotechnology and commercial
privacy.
c. The U.S. has the largest surplus with the Netherlands and the greatest deficit with
Japan.
Chapter 33: International Dimensions of Monetary and Fiscal Policies
Questions for Thought and Review
1.
At the time that this was written, it was believed that the dollar had risen high
enough. That is, the yen was as low as was desired. A higher yen against the dollar
would make imports more expensive, contributing to inflationary pressures, and
exports to Japan cheaper, exerting an expansionary effect on the economy. In the
early 2000s, the risk of inflation exceeded the risk of recession. Thus, one would not
desire a higher yen. A lower yen would have the desired effect of lowering
inflationary pressures and keeping the U.S. economy from overheating.
2.
At the time that this was written the U.S. trade deficit had risen to record highs. Still,
it is unclear whether we should want to lower the U.S. trade deficit. The trade deficit
was in part due to the fact that the economy has been growing for 9 consecutive
years. As long as the U.S. can borrow or sell assets, we can have a trade deficit. On
the other hand, the more we borrow, the more U.S. assets foreigners own. Eventually,
the U.S. will have to run a trade surplus.
Page 769, Answers to End-of-Chapter Questions
3.
If Japan ran an expansionary monetary policy, it would increase Japanese imports of
U.S. goods and thereby decrease the U.S. trade deficit. The U.S. dollar would rise
relative to the Japanese yen.
4.
A contractionary monetary policy by Japan would have the opposite effect as
described in question #3; the exchange rate on the U.S. dollar would fall, and the
U.S. trade deficit would worsen.
5.
Since the effect of monetary policy is to push the exchange rate down in all effects,
this will not change the effect presented in the chapter, other than to eliminate the
effect through income and replace it with the effect through prices.
6.
The effect of expansionary fiscal policy on the exchange rate is ambiguous, while
contractionary monetary policy has the effect of increasing exchange rates. The net
effect will depend on which influence is stronger.
7.
The answer to this question hinges on what is meant by "justified." If that means that
the United States is complaining about the actual negative consequences it
experiences because of this policy, as a matter of fact, one can say the complaint is
justified. If the argument centers on fairness, the issue is clearly complicated by the
question of whether a nation should put its goals ahead of or secondary to
international goals.
8.
In the 1990s one international goal of Germany was the European monetary system
and a common European currency. Domestically, one goal was keeping inflation low.
To keep inflation low, the Bundesbank followed tight monetary policy. This action,
however, boosted interest rates and the German exchange rate, which conflicted the
international goal of monetary union.
9.
If the recession was caused by a fall in domestic expenditures, we would expect that
its trade balance was falling. If, however, the recession was caused by a fall in
exports, we would expect that its trade balance was rising. The G-7 countries were
trying to get Japan to boost its economy by increasing aggregate expenditures and
expansionary monetary policy.
10.
When the U.S. debt in internationalized, there is a capital inflow into the U.S. Thus
the U.S. can consume more than it produces. Domestic borrowing would cause
crowding out but when foreigners make the loans that does not happen.
11.
The costs of internationalizing the debt is that interest and profits must be paid on the
foreign capital. Future consumption must be reduced to pay that amount.
Problems and Exercises
1.
See page 759 in the textbook.
Page 770, Answers to End-of-Chapter Questions
2.
See page 757 in the textbook.
3. a. A decrease in a country's competitiveness and an increase in the trade deficit is
probably due to expansionary fiscal policy which would increase inflation, reducing
competitiveness, and increase income, increasing imports and increasing the trade
deficit. Expansionary monetary policy would make the trade deficit larger, but its
effect on competitiveness through the exchange rate is ambiguous.
b. If interest rates have risen steadily along with a rise in the exchange rate, it is likely
that fiscal policy has been very expansionary.
c. Running more contractionary fiscal policy and expansionary monetary policy would
reduce the interest rate and thereby push down the exchange rate making the country
more competitive, while maintaining a constant domestic macro policy.
4. a. We would suggest that the IMF require both a contractionary monetary and fiscal
policy. I would, however, suggest a relatively more contractionary fiscal policy so
that the exchange rate would also fall, while inflation falls, boosting exports.
b This would tend to slow inflation, after an initial burst due to a fall in the exchange
rate. This, however, would hinder growth and push the economy into a recession.
c. We suspect that the country would not be happy about the proposal because its
adoption might lead to a deep recession, which is politically unpopular.
5.
a.
b.
c.
d.
The first advice I would give would be to explain that at most, I can talk about
tendencies rather than achieving goals. Not all goals are simultaneously achievable.
That advice given, I would provide the following recommendations:
We would suggest a contractionary fiscal policy, which lowers inflation and the
interest rate directly and reduces the trade deficit by lowering income. It will also
increase the capital inflow, which will tend to allow an increase in the trade deficit.
This would require a combination of monetary and fiscal policies—contractionary
monetary policy to head off inflation and increase interest rates and increase the value
of the dollar to encourage imports and discourage exports to reduce the trade deficit.
To further decrease the trade deficit, a slight expansionary fiscal policy could be
implemented, but that will have a tendency to push the interest rate up.
An expansionary monetary policy will reduce interest rates, depressing the dollars,
which will tend to reduce the trade deficit. This will tend to reduce unemployment
too. However, the increase in income and prices will tend to increase the trade deficit.
This combination of goals is difficult to achieve. Expansionary fiscal policy will tend
to reduce unemployment and increase interest rates. But to offset the effect of higher
income on increasing the trade deficit, an expansionary monetary policy will have to
be implemented to depress the dollar and spur exports to lower the trade deficit.
Reducing employment and the interest rate, this works in the opposite direction from
expansionary fiscal policy by reducing employment and the interest rate. Life is
tough.
Web Questions
Page 771, Answers to End-of-Chapter Questions
1. a.
b.
c.
d.
Canada, Mexico, Japan, Federal Republic of Germany, China.
Canada, Mexico, Japan, Federal Republic of Germany, China.
We would encourage them to follow expansionary monetary and fiscal policies.
Netherlands, Australia, Hong Kong, Belgium and Switzerland.
2. a. Conditionality is the requirement by the IMF that its aid recipients follow certain
policies, or conditions, in order to receive aid.
b. Typical IMF financing preconditions include reducing government spending,
budget deficits, and foreign (external) debt, reducing the rate of money growth to
control inflation, raising real interest rates to market levels and removing barriers to
export growth. When implemented, these conditions lead in the short term to (a) a
devaluation of local currency, (b) a lower trade deficit and (c) domestic problems
including slower growth and unemployment.
c. Mexico, Russia, Pakistan, Thailand and South Korea are examples of countries that
have received IMF financing over the past five years.
d. Mission creep is the term used to describe the increasing influence of the IMF on
domestic policies of its aid recipients. In addition to the enforcement of financial
reform in exchange for aid, the IMF has been accused of advocating an agenda in
relation to geopolitics and international security, social safety nets, government
corruption, the environment and human rights.
Chapter 34: Tools, Rules and Policy: The Art of Macro Policy
1.
The straight production possibility curve represents
constant marginal opportunity cost. The bowed out
production possibility curve represents increasing
marginal opportunity cost.
Good Y
Questions for Thought and Review
Good X
2.
GDP rose by 4 percent.
3.
Classical growth theory emphasizes saving and investment, and concludes that
increases in the growth rate are limited in the long run due to diminishing marginal
returns. New growth theory emphasizes technology and sees increases in the growth
rate as unlimited due to increasing returns.
Page 772, Answers to End-of-Chapter Questions
4.
We would respond that obviously they are not using this book. This book emphasizes
that the financial health of the economy, not the deficit, is the key issue in budget
policy and that commitments can be made that do not show up as a deficit but which
pose significant future problems.
5.
Increase by $200.
6.
Depreciate because the real interest rate in the United States is –1 percent and the real
interest rate in the rest of the world is 1 percent. The demand for the dollar currency
will fall and the supply will rise.
7.
As with most economic answers, it depends. In this case it depends on what the
inflation rate is. If the inflation rate is about 50% they can both be correct; John is
talking about real rates; Bob is talking about nominal rates.
8.
Monetary policy is considered the more effective tool since politically fiscal policy
has proven almost impossible to use due to the time lag.
9.
Policies followed now affect expectations of future policies, and those expectations
can affect how the economy operates. By thinking about policy as a process, not a
one time event, these affects are taken into account.
10.
A policy regime is a contingent rule about how policy makers will respond to
different events; a policy is a one-time action that does not consider policy as a
process.
11.
Who knows? Clearly in the early 2000s the economy has operated at lower inflation
rates and unemployment rates than economists thought possible. In that sense it is a
new era. But most economists question whether the new era has begun and whether
the inflationary consequences of low unemployment are just around the corner.
12.
Three explanations for why the U.S. economy did no well in the late 1990s are: 1)
increases in technology; 2) international competition; and 3) the policies the U.S. has
followed that have reduced regulation and started a balanced budget regime.
13.
The trade deficit is a potential problem because it means that our prosperity is based
on the inflow of foreign capital. Foreigners are owning more and more of the assets
in the U.S. If that foreign capital dries up, it could being the new era to a sudden halt.
Some economists dismiss this argument, arguing that the inflow is due to the
investment opportunities in the U.S. and that these will not suddenly end.
Problems and Exercises
Page 773, Answers to End-of-Chapter Questions
S0
Price
1. a. The supply curve shifts to the right. Equilibrium
price falls and equilibrium quantity rises. The
supply curve shifts from S0 to S1, equilibrium
price falls fr om P0 to P1 and equilibrium quantity
rises from Q0 to Q1.
b. The supply curve shifts to the left. Equilibrium
price rises and equilibrium quantity falls. The
supply curve shifts from S1 to S0, equilibrium
price rises from P1 to P0 and equilibrium quantity
falls from Q1 to Q0.
S1
P0
P1
D0
Q0 Q1
Quantity
2. a. Since actual income is 400 lower than targeted income, we would suggest increasing
government spending by 40. Since the multiplier is 10 this would increase output by
400 assuming the multiplier model was the correct one.
b. The budget deficit would increase by 40. If taxes were dependent on income, then
this would be offset somewhat by new revenue.
3.
We would work backwards. Since we want to increase income by $200 million, and
the multiplier is 10, we want to increase investment by 20 million. That will require a
.20 (twenty basis points) fall in the interest rate, or a $10 million increase in the
money supply. Since the money multiplier is 3.33, we would have to increase
reserves by $3 million. Thus, we would issue a directive for the Fed open market
window to buy $3 million worth of government bonds.
4. a. Forgetting about the rise in the price level initially, we would want to increase
income by 1,000. Since the multiplier is 5, that would require increasing government
spending by 200. However, since that will move the economy to above potential
income, income will rise by less than 1,000.
b. Follow the same policy as in a and output will rise by 1,000.
c. See the graphs below. In the AS/AD model on the left, the AD curve shifts to the
right by 1,000 to point B. Since the price level rises above 5,500, the AS curve shifts
from AS0 to AS1 and output rises by less than the desired amount. This same
scenario is shown on the right. The AE curve shifts up by 200 and equilibrium
output rises to 6,000. As the price level rises, however, the AE curve shifts down to
AE2 until equilibrium output is 5,500.
Page 774, Answers to End-of-Chapter Questions
Potential
Output
C
P1
P0
AP
AS1
A
B
AS0
Aggregate Expenditures
Price
Le ve l
AE1
AE2
AE0
B
C
AD1
A
AD0
5,000
5,500
6,000
Re al Output
5,000
5,500
6,000
Re al Output
5. a. Since the money multiplier is 2.5, we would issue a directive for the Fed open market
window to buy 24 worth of government bonds.
b. We could have also reduced the discount rate and lowered the reserve requirement,
although by how much cannot be determined with the information given.
c. Based on the quantity theory we would predict that the price level would rise because
of the increase in the money supply.
6. a. Expansionary fiscal policy will increase the trade deficit and have an indeterminate
affect on the exchange rate.
b. Contractionary monetary policy will increase the exchange rate and decrease the trade
deficit.
c. This combination would most likely decrease the exchange rate, and have an
indeterminate effect on the trade balance. The actual effects depends on how the
relative strengths of the policies.
An increase in U.S. tariffs will reduce the
supply of dollars to buy im ports. This shifts
the supply curve for dollars from S0 to S1.
The dollar’s value rises and quantity sold falls.
S1
Price
7.
S0
P1
P0
D0
Q1 Q0
Quantity
Web Questions
1.
There are a number of these jokes, many poking fun at the assumptions need to
conclude that no one would ever find a $20 lying on the sidewalk.
Page 775, Answers to End-of-Chapter Questions
2. a.
Consumer spending was strong and met expectations. In labor markets, difficulty in
finding and retaining qualified employees remained a common problem. Wages
experienced expansionary pressures and increasing input prices were noted in almost
every region. Industrial activity was strong overall.
b. The most recent monetary policy action taken by the FOMC was a 50 basis point
increase in the discount rate to 6 percent.
b. The FOMC should take into account growth rates, unemployment and inflation in
any decision to change the discount rate. In this case, the effect of the last rate hike
remains to be seen. However if present conditions continue unchecked another hike
may follow.
3. a. Dollarization is the process by which a country would eliminate its own currency
and adopt the dollar as legal tender.
b. Upon dollarization of its economy, a country would have to forego the profit from
issuing currency and adjust its policies to account for the loss of independent
monetary policy.
c. Dollarization abroad would stabilize and expand export markets, thereby helping
U.S. workers and businesses. Dollarization would reduce currency risk, thereby
helping U.S. investors. It would strengthen foreign economies, thereby reducing the
need to use taxpayers' money to bail out countries due to sudden currency-related
economic problems.
c. Ecuador and East Timor have enacted legislation to dollarize, while Argentina
and El Salvador have expressed interest in the idea. These are all emerging markets
that share a desire to avoid sudden currency-related crisis and would benefit from
measures to lower inflation and interest rates.
Page 776, Answers to End-of-Chapter Questions